DreamTheaterForums.org Dream Theater Fan Site

General => General Discussion => Topic started by: bloop on April 17, 2011, 12:41:33 PM

Title: 48÷2(9+3) =
Post by: bloop on April 17, 2011, 12:41:33 PM
48÷2(9+3) = ?

Something that has been circulating around the internet recently (sometimes used as troll material due to inevitable shitstorms.) However, I'm interested to know DTF's thoughts on the correct answer. It seems simple enough but there seems to be debate over whether the answer is  2 or 288. Consider order of operations as well as implied multiplication.
Title: Re: 48÷2(9+3) =
Post by: lonestar on April 17, 2011, 12:43:36 PM
I would say 2, but I suck at math.
Title: Re: 48÷2(9+3) =
Post by: Adami on April 17, 2011, 12:44:55 PM
I don't know what implied multiplication means, but order of operations would say that the answer is 2.
Title: Re: 48÷2(9+3) =
Post by: Dimitrius on April 17, 2011, 12:46:41 PM
It's 2 by order of operations.
Title: Re: 48÷2(9+3) =
Post by: TheVoxyn on April 17, 2011, 12:47:06 PM
Also going for 2.
Title: Re: 48÷2(9+3) =
Post by: Dimitrius on April 17, 2011, 12:48:30 PM
I don't know what implied multiplication means, but order of operations would say that the answer is 2.
Implied multiplication is what people use to write 2a instead or 2 x a.
Title: Re: 48÷2(9+3) =
Post by: Adami on April 17, 2011, 12:49:13 PM
I don't know what implied multiplication means, but order of operations would say that the answer is 2.
Implied multiplication is what people use to write 2a instead or 2 x a.

Oh, then I have no idea how that would change the answer.
Title: Re: 48÷2(9+3) =
Post by: bloop on April 17, 2011, 12:49:54 PM
Implied multiplication would mean the 2(12) would take precedence over the 48/2.
48 ÷ 2(9+3)=
48 ÷ 2(12)=
48 ÷ 24=
2
Title: Re: 48÷2(9+3) =
Post by: glaurung on April 17, 2011, 12:51:08 PM
2? What? The order of operations (https://en.wikipedia.org/wiki/Order_of_operations) says that in this case you do parentheses first and then divide/multiply from left to right.

48÷2(9+3) = ?
48÷2x 12 =?
24x12=?
288
Title: Re: 48÷2(9+3) =
Post by: Dimitrius on April 17, 2011, 12:51:11 PM
Implied multiplication would mean the 2(12) would take precedence over the 48/2.
Order of operations also tell you this...
Title: Re: 48÷2(9+3) =
Post by: blackngold29 on April 17, 2011, 12:51:18 PM
Order of operations

Parenthesis
Exponents
Multiplication / Division
Addition / Subtraction

So,

9+3 = 12

48 / 2 * 12

Go from left to right because multiplication and division are interchangeable

48 / 2 = 24 * 12 = 288


Alternatively,

Order of operations

Parenthesis
Exponents
Multiplication / Division
Addition / Subtraction

So,

9+3 = 12

48 / 2 * 12

Go from right to left because multiplication and division are interchangeable

48 / 24 = 2


The real answer: I don't care.
Title: Re: 48÷2(9+3) =
Post by: ZBomber on April 17, 2011, 12:52:30 PM
I always thought division/multiplication steps were interchangable, you just go by whatever one is first in the list.

But, like blackngold, I really don't care.
Title: Re: 48÷2(9+3) =
Post by: Dimitrius on April 17, 2011, 12:52:49 PM
2? What? The order of operations (https://en.wikipedia.org/wiki/Order_of_operations) says that in this case you do parentheses first and then divide/multiply from left to right.

48÷2(9+3) = ?
48÷2x 12 =?
Isn't it suppose to be 48/2(12)?
Title: Re: 48÷2(9+3) =
Post by: bloop on April 17, 2011, 12:53:22 PM
If we consider order of operations in which both multiplication and division are simplified left to right it would pan out this way-
48 ÷ 2(9+3)=
48 ÷ 2(12)=
24(12)=
188
Title: Re: 48÷2(9+3) =
Post by: glaurung on April 17, 2011, 12:53:43 PM
2? What? The order of operations (https://en.wikipedia.org/wiki/Order_of_operations) says that in this case you do parentheses first and then divide/multiply from left to right.

48÷2(9+3) = ?
48÷2x 12 =?
Isn't it suppose to be 48/2(12)?

Isn't that the exact same thing?
Title: Re: 48÷2(9+3) =
Post by: Dimitrius on April 17, 2011, 12:54:33 PM
2? What? The order of operations (https://en.wikipedia.org/wiki/Order_of_operations) says that in this case you do parentheses first and then divide/multiply from left to right.

48÷2(9+3) = ?
48÷2x 12 =?
Isn't it suppose to be 48/2(12)?

Isn't that the exact same thing?
Well, going by order of operations (or rather how I've always use order of operations) that would make the result 2 and not 288.
Title: Re: 48÷2(9+3) =
Post by: glaurung on April 17, 2011, 12:57:13 PM
2? What? The order of operations (https://en.wikipedia.org/wiki/Order_of_operations) says that in this case you do parentheses first and then divide/multiply from left to right.

48÷2(9+3) = ?
48÷2x 12 =?
Isn't it suppose to be 48/2(12)?

Isn't that the exact same thing?
Well, going by order of operations (or going by how I've always use order of operations) that would make the result 2 and not 288.
I don't know, I thought those were just two different ways of writing 2 multiplied by twelve.

Jamesman, get in here.
Title: Re: 48÷2(9+3) =
Post by: ZBomber on April 17, 2011, 12:57:48 PM
48/2(12) would mean you are diving 48 by 24... 48/2 x 12 would be diving 48 by 2 and then multiplying by 12, I think.

GOD I HATE MATH
Title: Re: 48÷2(9+3) =
Post by: blackngold29 on April 17, 2011, 12:58:47 PM
So because dividing by x is the same as multiplying by 1/x.

We can say that 48/2(12) is like (1/24) * 48 which is 2.
Title: Re: 48÷2(9+3) =
Post by: Dimitrius on April 17, 2011, 12:59:10 PM
48/2(12) would mean you are diving 48 by 24... 48/2 x 12 would be diving 48 by 2 and then multiplying by 12, I think.
This is how I look at it too.
Title: Re: 48÷2(9+3) =
Post by: lonestar on April 17, 2011, 01:01:34 PM
All this arguing, and the chef who hasn't done math in fifteen years might havge the right answer?? :metal
Title: Re: 48÷2(9+3) =
Post by: glaurung on April 17, 2011, 01:02:54 PM
Jamesman, we need you!
Title: Re: 48÷2(9+3) =
Post by: yorost on April 17, 2011, 01:04:35 PM
Standard order of operations:

48÷2 X (9+3) = 48÷2 X (12) = 48÷2 X 12 = 24 X 12 = 288

The problem is that with the implied multiplication most people interpret it as 48÷(2 X (9+3))=2, a stronger grouping than than any written division and multiplication symbols.  However, implied multiplication isn't clearly defined in the standard order of operations, so is not really treated any different than other multiplication or division symbols, i.e. order is left to right.
Title: Re: 48÷2(9+3) =
Post by: mizzl on April 17, 2011, 01:07:52 PM
2? What? The order of operations (https://en.wikipedia.org/wiki/Order_of_operations) says that in this case you do parentheses first and then divide/multiply from left to right.

48÷2(9+3) = ?
48÷2x 12 =?
24x12=?
288
This is the correct answer.
Title: Re: 48÷2(9+3) =
Post by: mizzl on April 17, 2011, 01:13:49 PM
48/2(12) would mean you are diving 48 by 24... 48/2 x 12 would be diving 48 by 2 and then multiplying by 12, I think.
This is how I look at it too.
I do not know of this mathematical operation 'diving' of yours.
Title: Re: 48÷2(9+3) =
Post by: Ben_Jamin on April 17, 2011, 01:13:50 PM
Its written like this

48÷2(9+3)
48÷2(12)
24(12)
288

Since 2(12) is 2 x 12. You divide first then multiply because Division is first in this equation, and is interchangable with multiplying.

Put it into a calculator as its written and you get 288


Pretty simple problem. What does
Title: Re: 48÷2(9+3) =
Post by: Plasmastrike on April 17, 2011, 01:17:22 PM
2? What? The order of operations (https://en.wikipedia.org/wiki/Order_of_operations) says that in this case you do parentheses first and then divide/multiply from left to right.

48÷2(9+3) = ?
48÷2x 12 =?
24x12=?
288
Title: Re: 48÷2(9+3) =
Post by: yorost on April 17, 2011, 01:20:31 PM
Put it into a calculator as its written and you get 288
Not all calculators/languages necessarily interpret things the same.  Standard order of operations isn't exactly an enforced standard, which is why using parens when faced with potentially ambiguous operations like this is usually emphasized.
Title: Re: 48÷2(9+3) =
Post by: bloop on April 17, 2011, 01:22:16 PM
Put it into a calculator as its written and you get 288
(https://cdn2.knowyourmeme.com/i/000/112/837/original/16h6ja8.jpg?1302454815)
I'm not saying it isn't 288 though.

Title: Re: 48÷2(9+3) =
Post by: Edan the Man on April 17, 2011, 01:30:40 PM
Since when is 2x ever any different at all from 2*x? Granted, since High School I've rarely had to use any algebra, but I've never heard such a thing. ???
Title: Re: 48÷2(9+3) =
Post by: HarlequinForest on April 17, 2011, 01:35:51 PM
There's no consensus in the math community about whether implied multiplication takes precedence over regular multiplication or not.  Think about that before you try to argue that there is a "right" answer, because there isn't.
Title: Re: 48÷2(9+3) =
Post by: glaurung on April 17, 2011, 01:37:22 PM
There's no consensus in the math community about whether implied multiplication takes precedence over regular multiplication or not.  Think about that before you try to argue that there is a "right" answer, because there isn't.

Yes there is.

And that answer is 288.
Title: Re: 48÷2(9+3) =
Post by: Dimitrius on April 17, 2011, 01:37:40 PM
There's no consensus in the math community about whether implied multiplication takes precedence over regular multiplication or not.  Think about that before you try to argue that there is a "right" answer, because there isn't.

Yes there is.

And that answer is 288.
Wrong. It's 2.
Title: Re: 48÷2(9+3) =
Post by: Ridley on April 17, 2011, 01:39:45 PM
With multiplication and division you just go from left to right, so 288. Not difficult, really.
Title: Re: 48÷2(9+3) =
Post by: bloop on April 17, 2011, 01:40:26 PM
There's no consensus in the math community about whether implied multiplication takes precedence over regular multiplication or not.  Think about that before you try to argue that there is a "right" answer, because there isn't.
True imo.

You can approach this two separate ways.

w/o implied multiplication
48/x(9+3)=288
(48/x)*12=288 (left to right simplification. Parenthesis around 48/x because you have to do that part first.)
48/x=24 (divide by 12)
x=2
So x=2 when the equation is set equal to 288.


Implied multiplication
48/x(9+3)=2
48/12x=2
4/x=2
x=2
So x=2 when the equation is set equal to 2.



However, 288 is generally the more accepted answer because implied multiplication in terms of order of operations is not entirely clear.
Title: Re: 48÷2(9+3) =
Post by: bloop on April 17, 2011, 01:44:57 PM
Also I added a poll.
Title: Re: 48÷2(9+3) =
Post by: HarlequinForest on April 17, 2011, 01:49:27 PM
There's no consensus in the math community about whether implied multiplication takes precedence over regular multiplication or not.  Think about that before you try to argue that there is a "right" answer, because there isn't.

Yes there is.

And that answer is 288.

Quote from: https://mathforum.org/library/drmath/view/54341.html
So to answer your question, I think both answers can be considered
right - which means, of course, that the question itself is wrong.

Quote from: https://en.wikipedia.org/wiki/Order_of_operations
Some mathematicians hold that multiplication by juxtaposition (omitting the x sign, ex. 2(4+3) ) is a symbol of grouping. No fixed convention exists.

Quote from: https://www.purplemath.com/modules/orderops2.htm
This next example displays an issue that almost never arises but, when it does, there seems to be no end to the arguing.

    Simplify 16 ÷ 2[8 – 3(4 – 2)] + 1.

        16 ÷ 2[8 – 3(4 – 2)] + 1
            = 16 ÷ 2[8 – 3(2)] + 1
            = 16 ÷ 2[8 – 6] + 1
            = 16 ÷ 2[2] + 1   (**)
            = 16 ÷ 4 + 1
            = 4 + 1
            = 5

The confusing part in the above calculation is how "16 divided by 2[2] + 1" (in the line marked with the double-star) becomes "16 divided by 4 + 1", instead of "8 times by 2 + 1". That's because, even though multiplication and division are at the same level (so the left-to-right rule should apply), parentheses outrank division, so the first 2 goes with the [2], rather than with the "16 divided by". That is, multiplication that is indicated by placement against parentheses (or brackets, etc) is "stronger" than "regular" multiplication. Typesetting the entire problem in a graphing calculator verifies this hierarchy:

    (https://www.purplemath.com/modules/orderops/order12.gif)

Note that different software will process this differently; even different models of Texas Instruments graphing calculators will process this differently. In cases of ambiguity, be very careful of your parentheses, and make your meaning clear. The general consensus among math people is that "multiplication by juxtaposition" (that is, multiplying by just putting things next to each other, rather than using the "×" sign) indicates that the juxtaposed values must be multiplied together before processing other operations. But not all software is programmed this way, and sometimes teachers view things differently. If in doubt, ask!

(And please do not send me an e-mail either asking for or else proffering a definitive verdict on this issue. As far as I know, there is no such final verdict. And telling me to do this your way will not solve the issue!)
Title: Re: 48÷2(9+3) =
Post by: Dimitrius on April 17, 2011, 01:52:13 PM
I think he was just joking HF...
Title: Re: 48÷2(9+3) =
Post by: rumborak on April 17, 2011, 01:52:43 PM
It's just bad syntax. So, "no definite answer" from me.

rumborak
Title: Re: 48÷2(9+3) =
Post by: Fiery Winds on April 17, 2011, 01:55:11 PM
Implied multiplication being of higher precedence is more an exception rather than the rule, especially in the math classes I've been in as a student and teacher.  If you want to deviate from the standard Order of Operations, use parenthesis, that's why they're there. 

Quick overview of Order of Operations:

Level 1: Parenthesis   {}[]()
Level 2: Exponents  x2, sqrt(), logxy
Level 3: Multiplication and Division  x, *, /
Level 4: Addition and Subtraction +, -

Each operation on the same level has equal precedence, so you don't always do multiplication first, or addition first.  Left to right precedence when dealing with operations residing on the same level.
Title: Re: 48÷2(9+3) =
Post by: yorost on April 17, 2011, 02:01:02 PM
It's just bad syntax. So, "no definite answer" from me.

rumborak

If you know how what you're writing for interprets it you're fine. :p  ...but yeah, it can't be said enough to not skip out on parens when writing code or expressions.  Even if you know how it is interpreted someone else reading your code in the future might get confused.  Exponents, negations, and functions are also big ones where it is often good to just be clear with your parens.
Title: Re: 48÷2(9+3) =
Post by: rumborak on April 17, 2011, 02:03:08 PM
IMHO, the "÷" sign ios something that is best left in 5th grade and below. Same thing with the forward-slash to indicate division.

rumborak
Title: Re: 48÷2(9+3) =
Post by: mizzl on April 17, 2011, 02:05:56 PM
IMHO, the "÷" sign ios something that is best left in 5th grade and below. Same thing with the forward-slash to indicate division.

rumborak

This. Real men use fractions.
Title: Re: 48÷2(9+3) =
Post by: Fiery Winds on April 17, 2011, 02:07:25 PM
IMHO, the "÷" sign ios something that is best left in 5th grade and below. Same thing with the forward-slash to indicate division.

rumborak


I dislike those signs as well, but they are a necessity when communicating via text.
Title: Re: 48÷2(9+3) =
Post by: rumborak on April 17, 2011, 02:14:17 PM
IMHO, the "÷" sign ios something that is best left in 5th grade and below. Same thing with the forward-slash to indicate division.

rumborak


I dislike those signs as well, but they are a necessity when communicating via text.

The forward-slash, yes (e.g. "5/(3+1)"), but the "÷" sign should IMHO never be used outside of the plain "X divided by Y".

rumborak
Title: Re: 48÷2(9+3) =
Post by: rumborak on April 17, 2011, 02:16:41 PM
IMHO, the "÷" sign ios something that is best left in 5th grade and below. Same thing with the forward-slash to indicate division.

rumborak

This. Real men use fractions.

Speaking of which, one of those "cultural differences" between Europeans and Americans is that Europeans love decimal writing, whereas Americans seem to hate it. There's the "5 and 3/16th of an inch" and the "$3 7/10" as the gas price, where I'm looking at it, thinking "wouldn't it have been easier to just write $3.70?".

rumborak
Title: Re: 48÷2(9+3) =
Post by: mizzl on April 17, 2011, 02:20:33 PM
IMHO, the "÷" sign ios something that is best left in 5th grade and below. Same thing with the forward-slash to indicate division.

rumborak

This. Real men use fractions.

Speaking of which, one of those "cultural differences" between Europeans and Americans is that Europeans love decimal writing, whereas Americans seem to hate it. There's the "5 and 3/16th of an inch" and the "$3 7/10" as the gas price, where I'm looking at it, thinking "wouldn't it have been easier to just write $3.70?".

rumborak

Really? I absolutely despise decimal writing. It's so imprecise!
Title: Re: 48÷2(9+3) =
Post by: yorost on April 17, 2011, 02:21:23 PM
Never seen $3 7/10 at a gas station in the US or Canada.  Are you sure you aren't seeing fractions of a cent?  I do see a lot of $3.45 9/10, but never fractions of dolars.
Title: Re: 48÷2(9+3) =
Post by: bloop on April 17, 2011, 02:22:48 PM
IMHO, the "÷" sign ios something that is best left in 5th grade and below. Same thing with the forward-slash to indicate division.

rumborak

It's not as if  the ÷ is ambiguous, as it's the same as /. However I understand what you mean, and if the original equation was in the form of a fraction there wouldn't be anything debatable about it.
(https://i559.photobucket.com/albums/ss37/ibloop/2.png)
For 2.
(https://i559.photobucket.com/albums/ss37/ibloop/1.png)
For 288.
Title: Re: 48÷2(9+3) =
Post by: rumborak on April 17, 2011, 02:23:29 PM
Never seen $3 7/10 at a gas station in the US or Canada.  Are you sure you aren't seeing fractions of a cent?  I do see a lot of $3.45 9/10, but never fractions of dolars.

Yeah, sorry, bad example on my part :lol

Yes, it's fractions of a cent. But, what's wrong with "$3.452"? I mean, would people really be that confused by it?

rumborak
Title: Re: 48÷2(9+3) =
Post by: orcus116 on April 17, 2011, 02:24:49 PM
Honestly, yes. Only because Americans are so used to seeing two decimal places so adding another one would completely shatter certain peoples minds.
Title: Re: 48÷2(9+3) =
Post by: rumborak on April 17, 2011, 02:27:09 PM
What they do in Germany a lot is to have the third digit be slightly different:

(https://www.merkur-online.de/bilder/2009/07/06/388402/379515119-schauplatz-eines-brutalen-verbrechens-wurde-jet-tankstelle-kassierer-wurde-beim-ueberfall-schwerst-v.9.jpg)
Title: Re: 48÷2(9+3) =
Post by: TimmyHiggy on April 17, 2011, 02:34:03 PM
My conclusion: The person that wrote the question did a piss poor job of it. You will never see maths published with ambiguous notation ever, and if someone had asked you to do that you would ask them what they meant. And I'm with everyone else who said that you are better off writing divisions as fractions, the primary school notation is best phased out as quickly as possible!
Title: Re: 48÷2(9+3) =
Post by: LieLowTheWantedMan on April 17, 2011, 02:53:12 PM
It's 288.
Title: Re: 48÷2(9+3) =
Post by: Jamesman42 on April 17, 2011, 03:10:58 PM
Hmm, that is tricky. So tricky because it looks like shoddy notation/writing rather than some arithmetic black hole. To explain, a THIRD possible answer is to distribute the 2 onto (9 + 3). So this is what happens:

48 ÷ 2(9 + 3)   [Given, with spaces added, which is allowed]
=48 ÷ 18 + 6    [Distribute the 2 onto the parentheses to get rid of them]
=8/3 + 6          [48/18 + 6]
=26/3

:biggrin:

But here is my real answer. I believe it is 288, because 48/2 is happening outside of the parentheses, but still to each other. Like, instead of using that elementary division sign, you are allowed to replace it with the forward slash, to get 48/2(9+3). Now if we distribute this number like I did in my joke math up there, we get (48/2)*9 + (48/2)*3 = 216 + 72 = 288. Q.E.D.lol.

All that said, I'm gonna ask some people tomorrow about this. Math professors and math majors alike. They are my people.


EDIT: I think the answer would be 2 if the 2 in the problem were included solely with the (9 + 3)...but then that would be obvious.
Title: Re: 48÷2(9+3) =
Post by: LearningToLive on April 17, 2011, 03:21:09 PM
The answer is 2. What kind of moron would say that there is no definitive "correct answer"?  :facepalm:
Title: Re: 48÷2(9+3) =
Post by: LieLowTheWantedMan on April 17, 2011, 03:21:53 PM
The answer is 288. :P

At least the way I'm being taught in school.
Title: Re: 48÷2(9+3) =
Post by: Jamesman42 on April 17, 2011, 03:25:13 PM
The answer is 2. What kind of moron would say that there is no definitive "correct answer"?  :facepalm:

And what kind of person comes onto a message board, calls some people a moron, gives an answer to a tricky question and does not back it up?
Title: Re: 48÷2(9+3) =
Post by: Dimitrius on April 17, 2011, 03:25:29 PM
The answer is 288. :P

At least the way I'm being taught in school.
Your school is obviously wrong.
Title: Re: 48÷2(9+3) =
Post by: tri.ad on April 17, 2011, 03:27:46 PM
The syntax doesn't make it clear, so there's no definitive answer to this imo.
Title: Re: 48÷2(9+3) =
Post by: LearningToLive on April 17, 2011, 03:28:39 PM
The answer is 2. What kind of moron would say that there is no definitive "correct answer"?  :facepalm:

And what kind of person comes onto a message board, calls some people a moron, gives an answer to a tricky question and does not back it up?
The kind of person that doesn't feel the need to repeat what several people have already said  :\

It's a mathematical equation. There is one answer to it.
Title: Re: 48÷2(9+3) =
Post by: millahh on April 17, 2011, 03:29:07 PM
The answer is 2. What kind of moron would say that there is no definitive "correct answer"?  :facepalm:

People who actually think about the question rather than being knee-jerk about the issue?
Title: Re: 48÷2(9+3) =
Post by: millahh on April 17, 2011, 03:30:21 PM
The answer is 2. What kind of moron would say that there is no definitive "correct answer"?  :facepalm:

And what kind of person comes onto a message board, calls some people a moron, gives an answer to a tricky question and does not back it up?
The kind of person that doesn't feel the need to repeat what several people have already said  :\

It's a mathematical equation. There is one answer to it.

There's one answer to a correctly written, unambiguous equation.
Title: Re: 48÷2(9+3) =
Post by: yorost on April 17, 2011, 03:33:46 PM
It's a mathematical equation. There is one answer to it.
You really missed the problem, didn't you?  Equations have to be parsed.
Title: Re: 48÷2(9+3) =
Post by: LearningToLive on April 17, 2011, 03:34:14 PM
/troll
Title: Re: 48÷2(9+3) =
Post by: Jamesman42 on April 17, 2011, 03:38:29 PM
Alright alright...here is another way to look at this. Division and multiplication are in the same rank, and an expression must be read left to right when operations are of the same rank.

48 ÷ 2(9 + 3) [Given]

= 48 ÷ 2 x (9+3) [Multiplication sign added to further show it is multiplication]

= 48 ÷ 2 x (12) [Parentheses, dogg]

= 24 x 12 [Read it left to right] [Smart]

= 288



Edit: Equations need an equal sign.
Title: Re: 48÷2(9+3) =
Post by: ScioPath on April 17, 2011, 03:41:07 PM
In order for a problem to have a an answer, it first must be a problem. And this problem sucks at being a problem.
Title: Re: 48÷2(9+3) =
Post by: LieLowTheWantedMan on April 17, 2011, 03:41:53 PM
Alright alright...here is another way to look at this. Division and multiplication are in the same rank, and an expression must be read left to right when operations are of the same rank.

48 ÷ 2(9 + 3) [Given]

= 48 ÷ 2 x (9+3) [Multiplication sign added to further show it is multiplication]

= 48 ÷ 2 x (12) [Parentheses, dogg]

= 24 x 12 [Read it left to right] [Smart]

= 288


Exactly.
Title: Re: 48÷2(9+3) =
Post by: Jamesman42 on April 17, 2011, 03:43:44 PM
In order for a problem to have a an answer, it first must be a problem. And this problem sucks at being a problem.
I guess the problem is that it sucks at being a problem...which is itself a problem, recursively becoming more and more problematic. Fibonacci headache.
Title: Re: 48÷2(9+3) =
Post by: Implode on April 17, 2011, 03:46:55 PM
It's just bad syntax. So, "no definite answer" from me.

rumborak

IMHO, the "÷" sign ios something that is best left in 5th grade and below. Same thing with the forward-slash to indicate division.

rumborak


I agree everything rumborak has said.
Title: Re: 48÷2(9+3) =
Post by: jsem on April 17, 2011, 03:48:41 PM
Except that...

2(9+3) =
(9x2 + 3x2) =
(18+6) =
24
Title: Re: 48÷2(9+3) =
Post by: Jamesman42 on April 17, 2011, 03:50:40 PM
That expression is true. But putting a "48 ÷" before it forces you to read it left to right starting with the 48.
Title: Re: 48÷2(9+3) =
Post by: rumborak on April 17, 2011, 03:50:42 PM
The answer is 2. What kind of moron would say that there is no definitive "correct answer"?  :facepalm:

And what kind of person comes onto a message board, calls some people a moron, gives an answer to a tricky question and does not back it up?
The kind of person that doesn't feel the need to repeat what several people have already said  :\

It's a mathematical equation. There is one answer to it.

Tell me, oracle boy, what happened in the sentence "the old lady hit the man with an umbrella"?

rumborak
Title: Re: 48÷2(9+3) =
Post by: ScioPath on April 17, 2011, 04:10:33 PM
That expression is true. But putting a "48 ÷" before it forces you to read it left to right starting with the 48.

Nope. Since the addition operation was in the parentheses, it must be done before the division.
Title: Re: 48÷2(9+3) =
Post by: Ħ on April 17, 2011, 04:13:51 PM
That expression is true. But putting a "48 ÷" before it forces you to read it left to right starting with the 48.

Nope. Since the addition operation was in the parentheses, it must be done before the division.
Yes, the parenthetical addition must be done first, but the mulitplication with 2 is not a correct step according to the OoO.
Title: Re: 48÷2(9+3) =
Post by: Jamesman42 on April 17, 2011, 04:15:37 PM
If you simplify the inside first (like you are supposed to), you get 12. So then it is read 48 ÷ 2(12). Division and multiplication are of the same rank, so when that happens, you read it left to right. So 48 ÷ 2 first, which is 24, and THEN multiply by 12, to get 288.
Title: Re: 48÷2(9+3) =
Post by: ScioPath on April 17, 2011, 04:22:56 PM
In order for a problem to have a an answer, it first must be a problem. And this problem sucks at being a problem.
I guess the problem is that it sucks at being a problem...which is itself a problem, recursively becoming more and more problematic. Fibonacci headache.

(https://www.wordans.us/wordansfiles/images/2011/1/18/63050/63050_popup.jpg?1295376369)
Title: Re: 48÷2(9+3) =
Post by: XJDenton on April 17, 2011, 04:24:49 PM
According to Wolfram Alpha its 288. The main point this example makes however is not to use the divide sign when you can help it.
Title: Re: 48÷2(9+3) =
Post by: bloop on April 17, 2011, 04:25:11 PM
In order for a problem to have a an answer, it first must be a problem. And this problem sucks at being a problem.
I guess the problem is that it sucks at being a problem...which is itself a problem, recursively becoming more and more problematic. Fibonacci headache.

(https://www.wordans.us/wordansfiles/images/2011/1/18/63050/63050_popup.jpg?1295376369)

:lol
Title: Re: 48÷2(9+3) =
Post by: robwebster on April 17, 2011, 04:37:25 PM
I only did maths up to A-Level standard - no university stuff, so I'm no expert - but I was always taught BODMAS. Brackets onto Division, Multiplication, Addition and Subtraction - in that order. The implicit hierarchy.

Which probably means my knowledge of maths is incomplete, seeing as everyone else is saying it's on level pegging with multiplication, but hey.

I do naturally convert it into 48 / 2(9+3) in my head, plus I tend to expand brackets first just to make things nice and easy, so I got 2, too. In spite of the hierarchy.
Title: Re: 48÷2(9+3) =
Post by: tjanuranus on April 17, 2011, 04:41:32 PM
Obviously 288 wtf?
Title: Re: 48÷2(9+3) =
Post by: kári on April 17, 2011, 04:43:17 PM
By order of operations, this means divide 48 by 2, and then multiply it by (9+3) which comes to 288. However in most math books and courses it is written wrongly for the sake of clarity. 1/2pi to me looks like 1 divided by 2pi but actually it's one half of pi. The correct notation here would be 1/(2pi).  It's just that sometimes the parentheses left out for reasons of speed or ease or whatever.
Title: Re: 48÷2(9+3) =
Post by: lonestar on April 17, 2011, 04:47:18 PM
All I got now is a fucking headache, thanks guys.
Title: Re: 48÷2(9+3) =
Post by: jsem on April 17, 2011, 04:49:15 PM
I get it nao lol.

I automatically jumped to this conclusion:

    48
______
2(9+3)

In which case it's 2. This is the same as 48 ÷ (2(9+3)). But that was not the given though.

Title: Re: 48÷2(9+3) =
Post by: lonestar on April 17, 2011, 04:51:56 PM
MAKE IT STOP!!! :zeltar:
Title: Re: 48÷2(9+3) =
Post by: Jamesman42 on April 17, 2011, 05:02:22 PM
I get it nao lol.

I automatically jumped to this conclusion:

    48
______
2(9+3)

In which case it's 2. This is the same as 48 ÷ (2(9+3)). But that was not the given though.



Yes, that is the initial conclusion, based on what kari rightfully said. This is why I always use extra parentheses to indicate what I mean when I am writing mathematics, because I want to leave no room for ambiguity.
Title: Re: 48÷2(9+3) =
Post by: ricky on April 17, 2011, 05:10:35 PM
What they do in Germany a lot is to have the third digit be slightly different:

(https://www.merkur-online.de/bilder/2009/07/06/388402/379515119-schauplatz-eines-brutalen-verbrechens-wurde-jet-tankstelle-kassierer-wurde-beim-ueberfall-schwerst-v.9.jpg)

holy shit. and i thought gas was expensive here.

and in europe, they go by the liter right? which is less than a gallon.
Title: Re: 48÷2(9+3) =
Post by: Crow on April 17, 2011, 05:13:49 PM
The way it's written in the OP, the problem is as follows:
48/2(9+3)
The first thing that happens is:
9+3 = 12
THEN.
48/2 = 24
And 24 * 12 = 288.

If it was written as such:
__48__
2(9+3)

The answer would be 2.
It is not written this way. The answer is 288.
Title: Re: 48÷2(9+3) =
Post by: Dr. DTVT on April 17, 2011, 05:43:01 PM
What they do in Germany a lot is to have the third digit be slightly different:

(https://www.merkur-online.de/bilder/2009/07/06/388402/379515119-schauplatz-eines-brutalen-verbrechens-wurde-jet-tankstelle-kassierer-wurde-beim-ueberfall-schwerst-v.9.jpg)

holy shit. and i thought gas was expensive here.

and in europe, they go by the liter right? which is less than a gallon.

But then you have to factor in exchange rate.  But if those are in Euros then its easily more expensive than even expensive American fuel.  And IIRC, gas is about $2-4 (US) more expensive than it is in the US for some reason, maybe because we can produce some of our own gasoline as well.
Title: Re: 48÷2(9+3) =
Post by: ricky on April 17, 2011, 05:45:48 PM
What they do in Germany a lot is to have the third digit be slightly different:

(https://www.merkur-online.de/bilder/2009/07/06/388402/379515119-schauplatz-eines-brutalen-verbrechens-wurde-jet-tankstelle-kassierer-wurde-beim-ueberfall-schwerst-v.9.jpg)

holy shit. and i thought gas was expensive here.

and in europe, they go by the liter right? which is less than a gallon.

But then you have to factor in exchange rate.  But if those are in Euros then its easily more expensive than even expensive American fuel.  And IIRC, gas is about $2-4 (US) more expensive than it is in the US for some reason, maybe because we can produce some of our own gasoline as well.

I know right, its crazy

but one thing i never understood is that in England (or Great britain, the UK, or whatever the proper term is, im still confused about that) is a part ot the EU. but they don't use the Euro. Why?
Title: Re: 48÷2(9+3) =
Post by: XJDenton on April 17, 2011, 05:51:36 PM
Because we are a bunch of euroskeptics who will have to be dragged in kicking and screaming.

In seriousness, there are 10 EU states that do not use the euro, so we aren't alone.
Title: Re: 48÷2(9+3) =
Post by: ricky on April 17, 2011, 05:53:00 PM
Because we are a bunch of euroskeptics who will have to be dragged in kicking and screaming.

In seriousness, there are 10 EU states that do not use the euro, so we aren't alone.

really? i didn't know that.

im kinda confused though, is there a significant reason why?
Title: Re: 48÷2(9+3) =
Post by: XJDenton on April 17, 2011, 05:58:02 PM
Xenophobia mostly. Britain is stuck in an anti-europe sentiment for the most part.
Title: Re: 48÷2(9+3) =
Post by: hefdaddy42 on April 17, 2011, 05:58:34 PM
I do naturally convert it into 48 / 2(9+3) in my head, plus I tend to expand brackets first just to make things nice and easy, so I got 2, too. In spite of the hierarchy.
This is the way it looks to me, and to me it is obviously 2.  But I was an English major, which means that I don't care.
Title: Re: 48÷2(9+3) =
Post by: ricky on April 17, 2011, 05:59:54 PM
Xenophobia mostly. Britain is stuck in an anti-europe sentiment for the most part.

wait so it is Britain.

but wait again don't you mean england? im confused.
Title: Re: 48÷2(9+3) =
Post by: bloop on April 17, 2011, 06:12:23 PM
Great Britain( informally Britain)= England, Scotland, and Wales.
Great Britain is an island.
England is a country that is a part of the island that is Great Britain.
Title: Re: 48÷2(9+3) =
Post by: ricky on April 17, 2011, 06:18:17 PM
Great Britain( informally Britain)= England, Scotland, and Wales.
Great Britain is an island.
England is a country that is a part of the island that is Great Britain.

What about North Ireland. I'm pretty sure that fits in somewhere.
Title: Re: 48÷2(9+3) =
Post by: Dimitrius on April 17, 2011, 06:19:07 PM
Great Britain( informally Britain)= England, Scotland, and Wales.
Great Britain is an island.
England is a country that is a part of the island that is Great Britain.

What about North Ireland. I'm pretty sure that fits in somewhere.
It's part of Great Britain.
Title: Re: 48÷2(9+3) =
Post by: ricky on April 17, 2011, 06:20:57 PM
um, ok. one last question then.


whats the UK?
Title: Re: 48÷2(9+3) =
Post by: bloop on April 17, 2011, 06:24:11 PM
I don't think Northern Ireland is a part of Great Britain, however it is a part of the UK.

To answer your question, the UK is Great Britain and Northern Ireland. (Hence "The United Kingdom of Great Britain and Northern Ireland")
Title: Re: 48÷2(9+3) =
Post by: Adami on April 17, 2011, 06:37:48 PM
I don't think Northern Ireland is a part of Great Britain, however it is a part of the UK.

To answer your question, the UK is Great Britain and Northern Ireland. (Hence "The United Kingdom of Great Britain and Northern Ireland")

Correct, Great Brittan is the major land mass of Brittan, including England Wales and Scotland, a few islands and such. Ireland is separate.
Title: Re: 48÷2(9+3) =
Post by: ricky on April 17, 2011, 06:43:31 PM
they should just call everything by one name to keep it simple.


and they should use the euro like most other union countries do, since they are part of the EU.
Title: Re: 48÷2(9+3) =
Post by: slycordinator on April 17, 2011, 07:18:14 PM
The way it's written in the OP, the problem is as follows:
48/2(9+3)
The first thing that happens is:
9+3 = 12
THEN.
48/2 = 24
The whole point of this discussion is that many math texts and mathematicians state that implied multiplication occurs before any other operations and many others disagree. So it's not a given that you divide in the 2nd step.

Another example of this is that one could say x^2y = (x^2)*y or x^(2*y).
Title: Re: 48÷2(9+3) =
Post by: Dimitrius on April 17, 2011, 07:25:44 PM
Another example of this is that one could say x^2y = (x^2)*y or x^(2*y).
I'm really intrigued to know how that is anything else but x^(2*y).
Title: Re: 48÷2(9+3) =
Post by: ricky on April 17, 2011, 07:30:34 PM
what's confusing to me isn't the answer to this problem.

its people doing math in their spare time.

jk. kinda.
Title: Re: 48÷2(9+3) =
Post by: Jamesman42 on April 17, 2011, 07:30:48 PM
I would immediately read that as x^(2y) but my logic tells me that it is (x^2)(y)
Title: Re: 48÷2(9+3) =
Post by: slycordinator on April 17, 2011, 07:35:04 PM
Another example of this is that one could say x^2y = (x^2)*y or x^(2*y).
I'm really intrigued to know how that is anything else but x^(2*y).
The only way it is x^(2*y) is by saying "the parentheses are implied" or by saying "implied multiplication comes before all other operations" but the fact of the matter is that there are no parentheses in the x^2y and it is not a given that implied multiplication comes before the other operations.
Title: Re: 48÷2(9+3) =
Post by: Dimitrius on April 17, 2011, 07:38:58 PM
Another example of this is that one could say x^2y = (x^2)*y or x^(2*y).
I'm really intrigued to know how that is anything else but x^(2*y).
The only way it is x^(2*y) is by saying "the parentheses are implied" or by saying "implied multiplication comes before all other operations" but the fact of the matter is that there are no parentheses in the x^2y and it is not a given that implied multiplication comes before the other operations.
Huh, I see. I would immediately read that as x^(2*y) and not be able to see it any other way.
Title: Re: 48÷2(9+3) =
Post by: Jamesman42 on April 17, 2011, 07:39:10 PM
I typed x^2x into my graphing calculator and it graphed x^3, so it seems to be the same deal...read it left to right and with PEMDAS, not what is implied.
Title: Re: 48÷2(9+3) =
Post by: ricky on April 17, 2011, 07:39:56 PM
Ok, my only serious post in this thread. This is like a 6th grade problem.

So, here we go, ill make this simple.

48÷2(9+3). PEMDAS = Parenthesis, then exponents, then Multi, then division, addition, subtraction. In that order. and remember, ITS FROM LEFT TO RIGHT.

k. so.

1. Parenthesis = 48÷2(12)
2. No exponents
3. MULTIPLICATION = 48÷24
4. Then division = 2.

YOU GUYS FORGET THE MOST IMPORTANT RULE, ITS FROM LEFT TO RIGHT.

yep.
Title: Re: 48÷2(9+3) =
Post by: slycordinator on April 17, 2011, 07:47:07 PM
PEMDAS = Parenthesis, then exponents, then Multi, then division, addition, subtraction. In that order.
Wrong.
PEMDAS = Parenthesis, exponents, multiplication & division, addition & subtraction. Multiplication and division have the same precedence.

15/3*4 is not 15/12 but is 20.
Title: Re: 48÷2(9+3) =
Post by: ScioPath on April 17, 2011, 07:48:53 PM
Division is multiplication.
Title: Re: 48÷2(9+3) =
Post by: ricky on April 17, 2011, 07:50:35 PM
i spent part of my free time talking about math.

i fail.
Title: Re: 48÷2(9+3) =
Post by: slycordinator on April 17, 2011, 07:56:01 PM
You wouldn't be the first person to think that multiplication comes first before division. I was taught that way in third grade but our fourth grade teacher corrected it...
Title: Re: 48÷2(9+3) =
Post by: lonestar on April 17, 2011, 07:56:09 PM
Only DTF could argue four pages in six hours about a fucking math problem. :rollin
Title: Re: 48÷2(9+3) =
Post by: ricky on April 17, 2011, 07:56:53 PM
You wouldn't be the first person to think that multiplication comes first before division. I was taught that way in third grade but our fourth grade teacher corrected it...

and im correcting your fourth grade teacher.
Title: Re: 48÷2(9+3) =
Post by: ricky on April 17, 2011, 07:57:48 PM
Only DTF could argue four pages in six hours about a fucking math problem. :rollin

well, we spent part of that debating why england (or britain, at this point i dont care) doesn't use the euro.


which doesn't make sense, if youre part of the EU you should use the Euro.
Title: Re: 48÷2(9+3) =
Post by: tjanuranus on April 17, 2011, 08:07:23 PM
WTF since when does multiplication come before division? They are on the exact same level of importance. Whichever one is first is what you do. In this case it's division.
Title: Re: 48÷2(9+3) =
Post by: ricky on April 17, 2011, 08:09:53 PM
Where in PEMDAS does it mention importance?

parenthesis, exponents, muli, division, add, sub, IN THAT ORDER. FROM LEFT TO RIGHT.

theres nothing in pemdas that says "both mulit and division are of equal importance". thats merely interpretation.
Title: Re: 48÷2(9+3) =
Post by: Dimitrius on April 17, 2011, 08:12:02 PM
Where in PEMDAS does it mention importance?

parenthesis, exponents, muli, division, add, sub, IN THAT ORDER. FROM LEFT TO RIGHT.

theres nothing in pemdas that says "both mulit and division are of equal importance". thats merely interpretation.
You do whatever comes first, it's multiplication OR division, add OR substraction. One doesn't come first than the other, one isn't more important than the other.
Title: Re: 48÷2(9+3) =
Post by: slycordinator on April 17, 2011, 08:12:22 PM
and im correcting your fourth grade teacher.
Then I guess my math teachers in junior high, high school, and college (where I got a math degree) corrected both my former teacher and you.

As noted above, division *is* multiplication. Dividing by a number is the same as multiplying by the number's inverse.

But in case you don't believe me:
https://en.wikipedia.org/wiki/Order_of_operations
https://www.mathsisfun.com/operation-order-pemdas.html
Title: Re: 48÷2(9+3) =
Post by: ricky on April 17, 2011, 08:15:48 PM
Where in PEMDAS does it mention importance?

parenthesis, exponents, muli, division, add, sub, IN THAT ORDER. FROM LEFT TO RIGHT.

theres nothing in pemdas that says "both mulit and division are of equal importance". thats merely interpretation.
You do whatever comes first, it's multiplication OR division, add OR substraction. One doesn't come first than the other, one isn't more important than the other.

THERE IS NO OR. if there was or, it would be PoEoMoDoAoS.
Title: Re: 48÷2(9+3) =
Post by: ricky on April 17, 2011, 08:16:46 PM
and im correcting your fourth grade teacher.
Then I guess my math teachers in junior high, high school, and college (where I got a math degree) corrected both my former teacher and you.

As noted above, division *is* multiplication. Dividing by a number is the same as multiplying by the number's inverse.

But in case you don't believe me:
https://en.wikipedia.org/wiki/Order_of_operations
https://www.mathsisfun.com/operation-order-pemdas.html

division isn't multiplication.

here's why.

2/2 = 1
2 *2 = 4.

how is that the same?

EDIt = AND WIKIPEDIA ISNT A VALID SOURCE *college professor rant*. I shit on eggs is a perfect example.
Title: Re: 48÷2(9+3) =
Post by: bloop on April 17, 2011, 08:17:16 PM
oh boy.
Title: Re: 48÷2(9+3) =
Post by: Dimitrius on April 17, 2011, 08:18:24 PM
and im correcting your fourth grade teacher.
Then I guess my math teachers in junior high, high school, and college (where I got a math degree) corrected both my former teacher and you.

As noted above, division *is* multiplication. Dividing by a number is the same as multiplying by the number's inverse.

But in case you don't believe me:
https://en.wikipedia.org/wiki/Order_of_operations
https://www.mathsisfun.com/operation-order-pemdas.html

division isn't multiplication.

here's why.

2/2 = 1
2 *2 = 4.

how is that the same?
You apparently don't know what a number's inverse is...

2/2 = 1
2 * (1/2) = 1
Title: Re: 48÷2(9+3) =
Post by: slycordinator on April 17, 2011, 08:19:23 PM
He's trolling.
Title: Re: 48÷2(9+3) =
Post by: ricky on April 17, 2011, 08:20:07 PM
and im correcting your fourth grade teacher.
Then I guess my math teachers in junior high, high school, and college (where I got a math degree) corrected both my former teacher and you.

As noted above, division *is* multiplication. Dividing by a number is the same as multiplying by the number's inverse.

But in case you don't believe me:
https://en.wikipedia.org/wiki/Order_of_operations
https://www.mathsisfun.com/operation-order-pemdas.html

division isn't multiplication.

here's why.

2/2 = 1
2 *2 = 4.

how is that the same?
You apparently don't know what a number's inverse is...

2/2 = 1
2 * (1/2) = 1

I realize that.

but were not talking about inverses.

were talking about multiplication and division.

its one thing to talk about the RELATION of the two, but the reality is THEY ARE SO DIFFERENT THINGS.
Title: Re: 48÷2(9+3) =
Post by: blackngold29 on April 17, 2011, 08:20:19 PM
Nobody addressed my point of diving by x is also the same as multiplying by 1/x

Therefore 48 * (1/24) = 2


Also, nowhere in the order of operations can I find "read it left to right." It's what I was taught, but you learn a lot of bullshit in school.


EDIT: ninja'd kind of
Title: Re: 48÷2(9+3) =
Post by: Jamesman42 on April 17, 2011, 08:20:48 PM
Where in PEMDAS does it mention importance?

parenthesis, exponents, muli, division, add, sub, IN THAT ORDER. FROM LEFT TO RIGHT.

theres nothing in pemdas that says "both mulit and division are of equal importance". thats merely interpretation.

You are dead wrong. You first do parentheses. Then exponents. Then multiplication/division, which, if there is more than one of those going on, you do it from left to right. Then addition/subtraction (same rule as multi/div.). PEMDAS is merely a mnemonic device to help you remember the order of the tiers, and it is NOT a literal order for ALL of the operations.

If you think I am even slightly wrong I will slap you across our monitors.
Title: Re: 48÷2(9+3) =
Post by: Jamesman42 on April 17, 2011, 08:22:03 PM
its one thing to talk about the RELATION of the two, but the reality is THEY ARE SO DIFFERENT THINGS.

Maybe you are not trolling, but division is technically multiplication.

When we divide something by 4, we are actually multiplying that something by (1/4). This may not make sense to you, but it is true.
Title: Re: 48÷2(9+3) =
Post by: ricky on April 17, 2011, 08:22:14 PM
Where in PEMDAS does it mention importance?

parenthesis, exponents, muli, division, add, sub, IN THAT ORDER. FROM LEFT TO RIGHT.

theres nothing in pemdas that says "both mulit and division are of equal importance". thats merely interpretation.

You are dead wrong. You first do parentheses. Then exponents. Then multiplication/division, which, if there is more than one of those going on, you do it from left to right. Then addition/subtraction (same rule as multi/div.). PEMDAS is merely a mnemonic device to help you remember the order of the tiers, and it is NOT a literal order for ALL of the operations.

If you think I am even slightly wrong I will slap you across our monitors.

yah but the poll says the answer is two.

and everyone in a dream theater forum is smart.

so yah.
Title: Re: 48÷2(9+3) =
Post by: Metro on April 17, 2011, 08:22:56 PM
I asked my cousin about this on Facebook. This was his response. Not sure sure this helps or not.

Quote
The order of operations dictates that everything in parentheses is done first, then multiplication/division, then addition/subtraction.

The misleading bit about this equation is that the easy assumption is that, since the 2 is adjacent to the "(9+3)," '2 x 12' should be done next. The reason for this is because we are taught to incorporate the distributive property -- a(b + c) = ab + ac -- without the important caveat that 'a(b + c)' should be viewed as 'a x (b + c)' when it exists in a longer equation such as the one listed above.

The most common answer to the equation in question is 2: 48 ÷ 2(9 + 3) = 48 ÷ (2 x 12) = 48 ÷ 24 = 2
This violates the procedural guidelines of order of operations.

The correct way to solve this equation is this:
48 ÷ 2(9 + 3) = (48 ÷ 2) x (9 + 3) = 24 x (9 + 3) = 24 x 12 = 288

I'm not sure what the debate is here. Putting a number adjacent to a parenthesis is simply shorthand for saying that the first number is to be multiplied by the second. There is not special consideration given to it just because it is written that way.

Hope this helps.
Title: Re: 48÷2(9+3) =
Post by: Jamesman42 on April 17, 2011, 08:23:43 PM
^That has been said a bunch of times already. ;)
Title: Re: 48÷2(9+3) =
Post by: Jamesman42 on April 17, 2011, 08:24:26 PM
Where in PEMDAS does it mention importance?

parenthesis, exponents, muli, division, add, sub, IN THAT ORDER. FROM LEFT TO RIGHT.

theres nothing in pemdas that says "both mulit and division are of equal importance". thats merely interpretation.

You are dead wrong. You first do parentheses. Then exponents. Then multiplication/division, which, if there is more than one of those going on, you do it from left to right. Then addition/subtraction (same rule as multi/div.). PEMDAS is merely a mnemonic device to help you remember the order of the tiers, and it is NOT a literal order for ALL of the operations.

If you think I am even slightly wrong I will slap you across our monitors.

yah but the poll says the answer is two.

and everyone in a dream theater forum is smart.

so yah.

Currently known, there are 24 people who can't do basic math. :biggrin:
Title: Re: 48÷2(9+3) =
Post by: ricky on April 17, 2011, 08:25:25 PM
Where in PEMDAS does it mention importance?

parenthesis, exponents, muli, division, add, sub, IN THAT ORDER. FROM LEFT TO RIGHT.

theres nothing in pemdas that says "both mulit and division are of equal importance". thats merely interpretation.

You are dead wrong. You first do parentheses. Then exponents. Then multiplication/division, which, if there is more than one of those going on, you do it from left to right. Then addition/subtraction (same rule as multi/div.). PEMDAS is merely a mnemonic device to help you remember the order of the tiers, and it is NOT a literal order for ALL of the operations.

If you think I am even slightly wrong I will slap you across our monitors.

yah but the poll says the answer is two.

and everyone in a dream theater forum is smart.

so yah.

Currently known, there are 24 people who can't do basic math. :biggrin:

yah but 24 > 14.

basic math says im right.
Title: Re: 48÷2(9+3) =
Post by: Jamesman42 on April 17, 2011, 08:26:22 PM
Trollllllllllllllllllllllllllll
Title: Re: 48÷2(9+3) =
Post by: Dimitrius on April 17, 2011, 08:27:27 PM
Trollllllllllllllllllllllllllll
His whole existence is that.
Title: Re: 48÷2(9+3) =
Post by: blackngold29 on April 17, 2011, 08:27:57 PM
Trollllllllllllllllllllllllllll
https://youtu.be/iwGFalTRHDA
Title: Re: 48÷2(9+3) =
Post by: Metro on April 17, 2011, 08:29:31 PM
^That has been said a bunch of times already. ;)

Eh. I figured it would have.
Title: Re: 48÷2(9+3) =
Post by: ricky on April 17, 2011, 08:30:05 PM
no no, i wasn't trolling. but technically, isn't calling someone a troll "trolling"? you can call me a troll, but i haven't resorted to namecalling, all ive done is make legitimate arguments.

the only thing i dont get is when people say multiplication and division are the same thing.
Title: Re: 48÷2(9+3) =
Post by: Jamesman42 on April 17, 2011, 08:32:45 PM
You are saying that, because 2 has the most votes, it must be right.

How is that a legitimate argument?
Title: Re: 48÷2(9+3) =
Post by: LieLowTheWantedMan on April 17, 2011, 08:33:43 PM
The answer is 288 because of BEDMAS. I hear most call it PEMDAS though. Maybe BEDMAS is the Canadian thing.
Title: Re: 48÷2(9+3) =
Post by: ricky on April 17, 2011, 08:34:14 PM
The answer is 288 because of BEDMAS. I hear most call it PEMDAS though. Maybe BEDMAS is the Canadian thing.

what does the b stand for?
Title: Re: 48÷2(9+3) =
Post by: Jamesman42 on April 17, 2011, 08:34:24 PM
Yeah, we use the word parentheses and y'all use brackets.
Title: Re: 48÷2(9+3) =
Post by: ricky on April 17, 2011, 08:34:51 PM
Yeah, we use the word parentheses and y'all use brackets.

ohhhhhh. my bad.

idk to me brackets makes more sense.
Title: Re: 48÷2(9+3) =
Post by: j on April 17, 2011, 08:38:05 PM
Did the math before entering the thread, also didn't read the thread.

The answer is 288.

/late

-J
Title: Re: 48÷2(9+3) =
Post by: Dimitrius on April 17, 2011, 08:38:42 PM
the only thing i dont get is when people say multiplication and division are the same thing.
When we divide something by 4, we are actually multiplying that something by (1/4). This may not make sense to you, but it is true.
Title: Re: 48÷2(9+3) =
Post by: ricky on April 17, 2011, 08:42:17 PM
the only thing i dont get is when people say multiplication and division are the same thing.
When we divide something by 4, we are actually multiplying that something by (1/4). This may not make sense to you, but it is true.

i just learned about this in my useless *math and physics theory* course.

saying "dividing something by 4" and saying "multiplying something by 1/4" are two completely different arguments. I really don't feel like copying and pasting the lecture.

Edit - I MEANT MATH APPLICATIONS COURSE.
Title: Re: 48÷2(9+3) =
Post by: lonestar on April 17, 2011, 08:47:24 PM
(https://t0.gstatic.com/images?q=tbn:ANd9GcSNapm6j7Ncfj3PyQphQVtnx7aHedD2-XaPoAJi3t9W7KYfNbdPTA)
Title: Re: 48÷2(9+3) =
Post by: Jamesman42 on April 17, 2011, 08:48:34 PM
the only thing i dont get is when people say multiplication and division are the same thing.
When we divide something by 4, we are actually multiplying that something by (1/4). This may not make sense to you, but it is true.

i just learned about this in my useless *math and physics theory* course.

saying "dividing something by 4" and saying "multiplying something by 1/4" are two completely different arguments. I really don't feel like copying and pasting the lecture.

Edit - I MEANT MATH APPLICATIONS COURSE.

You're right. Taking 14 math courses makes me wrong and you right. :)
Title: Re: 48÷2(9+3) =
Post by: ricky on April 17, 2011, 08:49:40 PM
the only thing i dont get is when people say multiplication and division are the same thing.
When we divide something by 4, we are actually multiplying that something by (1/4). This may not make sense to you, but it is true.

i just learned about this in my useless *math and physics theory* course.

saying "dividing something by 4" and saying "multiplying something by 1/4" are two completely different arguments. I really don't feel like copying and pasting the lecture.

Edit - I MEANT MATH APPLICATIONS COURSE.

You're right. Taking 14 math courses makes me wrong and you right. :)

i never said you were wrong. never in one of my posts did i say that.
Title: Re: 48÷2(9+3) =
Post by: ricky on April 17, 2011, 08:50:19 PM
(https://t0.gstatic.com/images?q=tbn:ANd9GcSNapm6j7Ncfj3PyQphQVtnx7aHedD2-XaPoAJi3t9W7KYfNbdPTA)

+1

me too. it makes me fight with jamesman who in reality i <3.
Title: Re: 48÷2(9+3) =
Post by: rumborak on April 17, 2011, 08:51:38 PM
I hate it when people overly rely on operator precedence, especially in programming languages.
Stuff like

int x = ++i * 5 + i;

makes me want to track down the originator of the code and "++" him myself a bit.

rumborak
Title: Re: 48÷2(9+3) =
Post by: lonestar on April 17, 2011, 08:54:28 PM
(https://cdn3.knowyourmeme.com/i/000/113/276/original/206483_1776491504605_1608886476_1688467_6580770_n.jpg?1302609835)
 :biggrin:
Title: Re: 48÷2(9+3) =
Post by: ricky on April 17, 2011, 08:56:33 PM
..and were back to the OP of this thread.
Title: Re: 48÷2(9+3) =
Post by: Implode on April 17, 2011, 08:58:19 PM
I'm surprised people are still arguing in this thread.  :|
Title: Re: 48÷2(9+3) =
Post by: Dimitrius on April 17, 2011, 09:03:50 PM
I'm surprised people are still arguing in this thread.  :|
Welcome to DTF.
Title: Re: 48÷2(9+3) =
Post by: ricky on April 17, 2011, 09:05:21 PM
well, two is still winning, so i guess that must be the answer.

and i officially give up.
Title: Re: 48÷2(9+3) =
Post by: XJDenton on April 17, 2011, 09:06:34 PM
EDIt = AND WIKIPEDIA ISNT A VALID SOURCE *college professor rant*. I shit on eggs is a perfect example.

This however is.

https://mathworld.wolfram.com/Precedence.html
Title: Re: 48÷2(9+3) =
Post by: Implode on April 17, 2011, 09:06:55 PM
I'm surprised people are still arguing in this thread.  :|
Welcome to DTF.

Maybe I should make a 1 = 0.999... thread. :neverusethis:
Title: Re: 48÷2(9+3) =
Post by: Jamesman42 on April 17, 2011, 09:10:18 PM
(https://t0.gstatic.com/images?q=tbn:ANd9GcSNapm6j7Ncfj3PyQphQVtnx7aHedD2-XaPoAJi3t9W7KYfNbdPTA)

+1

me too. it makes me fight with jamesman who in reality i <3.

Well, your assertions fly in the face of mathematical logic, so.....ya know. You're roughing me up, son
Title: Re: 48÷2(9+3) =
Post by: Jamesman42 on April 17, 2011, 09:10:57 PM
I'm surprised people are still arguing in this thread.  :|
Welcome to DTF.

Maybe I should make a 1 = 0.999... thread. :neverusethis:

Hehe

It's true though!
Title: Re: 48÷2(9+3) =
Post by: ricky on April 17, 2011, 09:12:22 PM
(https://t0.gstatic.com/images?q=tbn:ANd9GcSNapm6j7Ncfj3PyQphQVtnx7aHedD2-XaPoAJi3t9W7KYfNbdPTA)

+1

me too. it makes me fight with jamesman who in reality i <3.

Well, your assertions fly in the face of mathematical logic, so.....ya know. You're roughing me up, son

sorry, ill apologize in the chat thread tomorrow. you might have to remind me tho, k?

THAT REMINDS ME, ITS BEEN 36 HOURS. my avatar can go back to normal now.
Title: Re: 48÷2(9+3) =
Post by: Implode on April 17, 2011, 09:12:28 PM
Hehe

It's true though!

DON'T START. Though I agree.
Title: Re: 48÷2(9+3) =
Post by: Jamesman42 on April 17, 2011, 09:18:08 PM
1 = 1
1/9 = 1/9
1/9 = 0.11111...
9*(1/9) = 9*(0.11111...)
1 = .99999...

QED
Title: Re: 48÷2(9+3) =
Post by: rumborak on April 17, 2011, 09:19:04 PM
RPN shits on this thread. My first calculator in college had RPN, that was pretty badass.

rumborak
Title: Re: 48÷2(9+3) =
Post by: Implode on April 17, 2011, 09:20:25 PM
1 = 1
1/9 = 1/9
1/9 = 0.11111...
9*(1/9) = 9*(0.11111...)
1 = .99999...

QED

I prefer this: https://upload.wikimedia.org/math/6/f/a/6fa510b44742046a167b4b8515162825.png

And I wish I could change my vote to 288. Ah well.
Title: Re: 48÷2(9+3) =
Post by: Jamesman42 on April 17, 2011, 09:21:51 PM
^Nice, using infinite series to show it. I've never seen that version before, thank you
Title: Re: 48÷2(9+3) =
Post by: rumborak on April 17, 2011, 09:22:17 PM
1 = 1
1/9 = 1/9
1/9 = 0.11111...
9*(1/9) = 9*(0.11111...)
1 = .99999...

QED

The bolded part is kinda the weak part in the argument. You're essentially trying to prove this by relying on a convenient shorthand.

rumborak
Title: Re: 48÷2(9+3) =
Post by: Jamesman42 on April 17, 2011, 09:22:50 PM
What do you mean exactly?
Title: Re: 48÷2(9+3) =
Post by: Implode on April 17, 2011, 09:23:45 PM
I would kind of agree with rumby. The fact that 1/9 = 0.111... comes from the same logic that 1 = 0.999....
Title: Re: 48÷2(9+3) =
Post by: Jamesman42 on April 17, 2011, 09:27:33 PM
Then what does 1/9 equal in decimal form?
Title: Re: 48÷2(9+3) =
Post by: rumborak on April 17, 2011, 09:30:51 PM
What do you mean exactly?


I'm saying that the "..." notation isn't really a proper notation for numbers, and it kinda means "reader, I assume you know what I mean, and I'm too lazy to write infinite numbers here". Using this notation for proving something doesn't really work too well.

Then what does 1/9 equal in decimal form?

There is none. There are many numbers that don't have a decimal expansion, including Pi.

1 = 1
1/9 = 1/9
1/9 = 0.11111...
9*(1/9) = 9*(0.11111...)
1 = .99999...

QED
I prefer this: https://upload.wikimedia.org/math/6/f/a/6fa510b44742046a167b4b8515162825.png

This one isn't really that great either. It just relies on a different assumption, which is that the limit of x going towards a number is that number. So, in the end you have a tautology, since the limit notation relies on exactly the same thing as the thing you're trying to prove.

To me, it's mostly just a matter of definition that 0.99999... = 1. The argument goes that the difference is infinitely small and thus you could never define a meaningful difference between the two.

rumborak
Title: Re: 48÷2(9+3) =
Post by: Implode on April 17, 2011, 09:34:06 PM
Then what does 1/9 equal in decimal form?

0.111... but if that's the case you might as well cut out the first two steps in your proof. I dunno. I feel like it's cheating. I'm actually not an expert.

To me, it's mostly just a matter of definition that 0.99999... = 1. The argument goes that the difference is infinitely small and thus you could never define a meaningful difference between the two.

rumborak


I'm sure there are huge math texts that explain it, but I don't know how to respond. You're kind of right, I guess.
Title: Re: 48÷2(9+3) =
Post by: Fiery Winds on April 17, 2011, 09:47:52 PM
I'm saying that the "..." notation isn't really a proper notation for numbers, and it kinda means "reader, I assume you know what I mean, and I'm too lazy to write infinite numbers here". Using this notation for proving something doesn't really work too well.

It's just a way to represent the bar symbol to indicate a repeating decimal.

Quote
There are many numbers that don't have a decimal expansion, including Pi.

The difference is that Pi is an irrational number which cannot be represented as a ratio of two whole numbers.  A repeating decimal is still a decimal.

Quote
To me, it's mostly just a matter of definition that 0.99999... = 1. The argument goes that the difference is infinitely small and thus you could never define a meaningful difference between the two.

Some may argue that, but the proof is meant to show that they are in fact equal to each other. 

Title: Re: 48÷2(9+3) =
Post by: rumborak on April 17, 2011, 09:52:07 PM
Quote
The difference is that Pi is an irrational number which cannot be represented as a ratio of two whole numbers.  A repeating decimal is still a decimal.

My point is that neither 0.9999... nor 3.1415... describe through their notation the number correctly. Using a flawed notation to prove something leads to a weak argument. From Wikipedia:

Quote
William Byers argues that a student who agrees that 0.999... = 1 because of the above proofs, but hasn't resolved the ambiguity, doesn't really understand the equation.[2] Fred Richman argues that the first argument "gets its force from the fact that most people have been indoctrinated to accept the first equation without thinking".[3]

Quote
Some may argue that, but the proof is meant to show that they are in fact equal to each other.  

I am not disputing the truth of the statement. I'm saying the proofs need to be better than just the superficial proofs mentioned above.

rumborak
Title: Re: 48÷2(9+3) =
Post by: Jamesman42 on April 17, 2011, 09:54:44 PM
Rumby, the thing is, we KNOW that 1/9 = .11111... or .1 with the bar over it. It is a repeating decimal. Then, since we know that, by multiplying .11111....by 9, we can logically conclude that all those 1's turn into 9's, because (1)(9) = 9.

I agree that it is more of a definition, but these "proofs" are a good way to see why it's true.
Then what does 1/9 equal in decimal form?

0.111... but if that's the case you might as well cut out the first two steps in your proof. I dunno. I feel like it's cheating. I'm actually not an expert.

Eh, I just like to start with 1 = 1 in those proofs, it makes it more elegant to me. That's just me, though.


Edit: This thread has been burned. Also I put "nonrepeating" facepaaaaalm
Title: Re: 48÷2(9+3) =
Post by: Implode on April 17, 2011, 09:56:30 PM
And it's all my fault!  :'(










((https://files.sharenator.com/success_baby_asshole_Justin_Bieber_Hit_With_Water_Bottle-s462x338-84972-535.jpg))
Title: Re: 48÷2(9+3) =
Post by: lonestar on April 17, 2011, 09:57:00 PM
You guys have gone over my head so many times in this thread, I have no hair left up there.
Title: Re: 48÷2(9+3) =
Post by: Jamesman42 on April 17, 2011, 09:57:48 PM
You guys have gone over my head so many times in this thread, I have no hair left up there.

One divided by zero equals infinity.
Title: Re: 48÷2(9+3) =
Post by: Nic35 on April 17, 2011, 09:58:27 PM
How is it possible :huh:
Title: Re: 48÷2(9+3) =
Post by: King Postwhore on April 17, 2011, 09:58:47 PM
You guys have gone over my head so many times in this thread, I have no hair left up there.

Maybe if they explained in tbsp and tsp.............
Title: Re: 48÷2(9+3) =
Post by: Implode on April 17, 2011, 10:01:12 PM
(https://i11.photobucket.com/albums/a194/QWERTYkid911/Status.png)

 :corn
Title: Re: 48÷2(9+3) =
Post by: glaurung on April 17, 2011, 10:02:08 PM
You guys have gone over my head so many times in this thread, I have no hair left up there.

"Buffalo buffalo Buffalo buffalo buffalo buffalo Buffalo buffalo." is a grammatically valid sentence.
Title: Re: 48÷2(9+3) =
Post by: lonestar on April 17, 2011, 10:02:17 PM
You guys have gone over my head so many times in this thread, I have no hair left up there.

One divided by zero equals infinity.
(https://t1.gstatic.com/images?q=tbn:ANd9GcTKDMSdGPiM98oNCAT9zr599-9wmuW4G8EOS6I03Afma9_NF38k8A)
Title: Re: 48÷2(9+3) =
Post by: rumborak on April 17, 2011, 10:04:40 PM
Rumby, the thing is, we KNOW that 1/9 = .11111... or .1 with the bar over it. It is a repeating decimal. Then, since we know that, by multiplying .11111....by 9, we can logically conclude that all those 1's turn into 9's, because (1)(9) = 9.

Err, no, James, we DON'T know that 1/9 = .11111..., because it is just a rewording of what you are trying to prove!! (that 0.9999... = 1)
A tautology is not a proof.

rumborak
Title: Re: 48÷2(9+3) =
Post by: lonestar on April 17, 2011, 10:11:52 PM
(https://www.yesfans.com/images/smilies/horse.gif)
Title: Re: 48÷2(9+3) =
Post by: glaurung on April 17, 2011, 10:14:20 PM
:deadhorse:
Title: Re: 48÷2(9+3) =
Post by: lonestar on April 17, 2011, 10:15:10 PM
:deadhorse:
Huh.  Didn't know we had that one.
Title: Re: 48÷2(9+3) =
Post by: blackngold29 on April 17, 2011, 10:21:33 PM
I also posted this on facebook. Got two people that said 2, posted it step by step showing how to get 288. And then someone else answered 144. :justjen
Title: Re: 48÷2(9+3) =
Post by: lonestar on April 17, 2011, 10:48:12 PM
OK, last one of the night...
(https://t3.gstatic.com/images?q=tbn:ANd9GcT84zF6NG28uUxKZSQ6d2T3rjj1tF11iYl6qteV0eb9ugow1HYC)
Title: Re: 48÷2(9+3) =
Post by: Jamesman42 on April 17, 2011, 10:50:13 PM
Rumby, the thing is, we KNOW that 1/9 = .11111... or .1 with the bar over it. It is a repeating decimal. Then, since we know that, by multiplying .11111....by 9, we can logically conclude that all those 1's turn into 9's, because (1)(9) = 9.

Err, no, James, we DON'T know that 1/9 = .11111..., because it is just a rewording of what you are trying to prove!! (that 0.9999... = 1)
A tautology is not a proof.

rumborak


What does your calculator tell you then? I'm not getting your logic here.
Title: Re: 48÷2(9+3) =
Post by: kári on April 18, 2011, 02:18:18 AM
What they do in Germany a lot is to have the third digit be slightly different:

(https://www.merkur-online.de/bilder/2009/07/06/388402/379515119-schauplatz-eines-brutalen-verbrechens-wurde-jet-tankstelle-kassierer-wurde-beim-ueberfall-schwerst-v.9.jpg)

holy shit. and i thought gas was expensive here.

and in europe, they go by the liter right? which is less than a gallon.
Those prices are outdated.. Diesel is € 1,45 or so at the moment, and gasoline is about 1,65... That's $7,858 and $8,672 per gallon.
Title: Re: 48÷2(9+3) =
Post by: jsem on April 18, 2011, 02:20:39 AM
The reason gas is pricier in most european countries is because of taxes.
Title: Re: 48÷2(9+3) =
Post by: kári on April 18, 2011, 02:24:09 AM
Well of course... But that doesn't make it any better or does it?


Only DTF could argue four pages in six hours about a fucking math problem. :rollin
This thread reached 27 pages in 2 days on some other forum and was locked because there was too much flaming etc...
Title: Re: 48÷2(9+3) =
Post by: kári on April 18, 2011, 02:38:28 AM
Rumby, the thing is, we KNOW that 1/9 = .11111... or .1 with the bar over it. It is a repeating decimal. Then, since we know that, by multiplying .11111....by 9, we can logically conclude that all those 1's turn into 9's, because (1)(9) = 9.

Err, no, James, we DON'T know that 1/9 = .11111..., because it is just a rewording of what you are trying to prove!! (that 0.9999... = 1)
A tautology is not a proof.

rumborak


How about
x = 0.99...
10x = 9.99...
9x = 9.99... - x
9x = 9
x = 1
Title: Re: 48÷2(9+3) =
Post by: XJDenton on April 18, 2011, 03:09:01 AM
Rumby, the thing is, we KNOW that 1/9 = .11111... or .1 with the bar over it. It is a repeating decimal. Then, since we know that, by multiplying .11111....by 9, we can logically conclude that all those 1's turn into 9's, because (1)(9) = 9.

Err, no, James, we DON'T know that 1/9 = .11111..., because it is just a rewording of what you are trying to prove!! (that 0.9999... = 1)
A tautology is not a proof.

rumborak


Forgive me, but can't you arrive at 1/9 = 0.1... through simple long division?
Title: Re: 48÷2(9+3) =
Post by: kári on April 18, 2011, 03:20:48 AM
I think rumborak's point is that 0.11... isn't a correct notation for what you get when you divide 1 by 9. Or something like that.
Title: Re: 48÷2(9+3) =
Post by: XJDenton on April 18, 2011, 03:30:55 AM
Well it definitely is the correct notation for that fraction. I think Rumborak's point was that the proof was relying on two unproved axioms, each a different wording of the same problem, whereas in the case of 1/9, the answer of 0.111... can be found using simple mathematics (long division). Admittedly its not the best proof in the world but its not a tautology as Rumborak suggested.
Title: Re: 48÷2(9+3) =
Post by: RuRoRul on April 18, 2011, 04:48:32 AM
Well this thread has moved on quite a bit from when I saw it yesterday...

All I would say is (and all this has probably already been said) this is just poor notation, it should be written using fractions rather than the ÷ sign as it leads to the ambiguity.

Multiplication and division have the same priority, even though the common acronym to remember the order might put them one in front of the other (BODMAS or PEMDAS). But this is just a way of remembering it, there is no rule saying a direct multiplication should go before division because they are essentially the same thing. Going left to right technically leads to 288 as it is 48 * 1/2 * (9+3).

However, intuitively I would be inclined to say that the 2(9+3) is supposed to be grouped together because of the way it is written. The 2(9+3) represents (9+3) with a common factor of 2 taken outside, which is 24. Then it becomes 48 * 1/24 which is 2. But I see in this thread people saying that the 2(9+3) notation isn't accepted as a form of grouping any more than 2 x (9+3), so using a left to right convention would lead to 288.
Title: Re: 48÷2(9+3) =
Post by: Jamesman42 on April 18, 2011, 06:19:26 AM
Well it definitely is the correct notation for that fraction. I think Rumborak's point was that the proof was relying on two unproved axioms, each a different wording of the same problem, whereas in the case of 1/9, the answer of 0.111... can be found using simple mathematics (long division). Admittedly its not the best proof in the world but its not a tautology as Rumborak suggested.

This is my belief as well. I understand that the proof may have some shakiness to it, but it still works to support the definition that 0.999... = 1.
Title: Re: 48÷2(9+3) =
Post by: YtseBitsySpider on April 18, 2011, 06:41:28 AM
B - brackets
E - exponents
D - division
M - multiplication
A - addition
A - subtraction


BEDMAS.
Learn it.
Title: Re: 48÷2(9+3) =
Post by: Xanthul on April 18, 2011, 06:53:23 AM
48/2(9+3) = 24 * 12 = 288
48/(2(9+3)) = 48 / 24 = 2

That's what I've been taught and what I've seen in any math book I've ever used/read.
Title: Re: 48÷2(9+3) =
Post by: RuRoRul on April 18, 2011, 07:20:10 AM
Quote
B - brackets
E - exponents
D - division
M - multiplication
A - addition
A - subtraction

BEDMAS.
Learn it.

Yeah, except the whole point of this is that division and multiplication aren't done in any certain order the same way addition and subtraction aren't...

Also, that's BEDMAA.
Title: Re: 48÷2(9+3) =
Post by: rumborak on April 18, 2011, 08:01:47 AM
Well it definitely is the correct notation for that fraction. I think Rumborak's point was that the proof was relying on two unproved axioms, each a different wording of the same problem, whereas in the case of 1/9, the answer of 0.111... can be found using simple mathematics (long division). Admittedly its not the best proof in the world but its not a tautology as Rumborak suggested.

To elaborate on my point: In the end, 0.999... = 1 makes a statement about the infinitely small remainder between the two numbers, i.e. that the remainder *is* indeed zero and they are thus the same numbers.
Now, 1/9 = 0.1111.... makes exactly the same point! Just as in 0.9999... the argument goes there is always another "9" that makes it closer to 1, in 0.1111... there is always another 1 that makes it closer to 1/9.

So, my point is that you are starting out with the thing you are actually trying to prove, and then, duh, you magically prove it!

Again, I do not question the overall truth of the statement, I'm just commenting on that a lot of those "easy proofs" for this are inherently flawed.

rumborak
Title: Re: 48÷2(9+3) =
Post by: kári on April 18, 2011, 08:10:53 AM
Well it definitely is the correct notation for that fraction. I think Rumborak's point was that the proof was relying on two unproved axioms, each a different wording of the same problem, whereas in the case of 1/9, the answer of 0.111... can be found using simple mathematics (long division). Admittedly its not the best proof in the world but its not a tautology as Rumborak suggested.

To elaborate on my point: In the end, 0.999... = 1 makes a statement about the infinitely small remainder between the two numbers, i.e. that the remainder *is* indeed zero and they are thus the same numbers.
Now, 1/9 = 0.1111.... makes exactly the same point! Just as in 0.9999... the argument goes there is always another "9" that makes it closer to 1, in 0.1111... there is always another 1 that makes it closer to 1/9.

So, my point is that you are starting out with the thing you are actually trying to prove, and then, duh, you magically prove it!

Again, I do not question the overall truth of the statement, I'm just commenting on that a lot of those "easy proofs" for this are inherently flawed.

rumborak

Then what is the "real proof"? I think we proved it last semester in one of my courses called "Proving and reasoning" using Dedekindsneden, whatever that means in English.. Dedekind cuts?
Title: Re: 48÷2(9+3) =
Post by: blackngold29 on April 18, 2011, 08:12:59 AM
Quote
B - brackets
E - exponents
D - division
M - multiplication
A - addition
A - subtraction

BEDMAS.
Learn it.

Yeah, except the whole point of this is that division and multiplication aren't done in any certain order the same way addition and subtraction aren't...

Also, that's BEDMAA.
Correct. In school I was taught "Please Excuse My Dear Aunt Sally" which reverses division and multiplication.
Title: Re: 48÷2(9+3) =
Post by: rumborak on April 18, 2011, 08:16:19 AM
Well it definitely is the correct notation for that fraction. I think Rumborak's point was that the proof was relying on two unproved axioms, each a different wording of the same problem, whereas in the case of 1/9, the answer of 0.111... can be found using simple mathematics (long division). Admittedly its not the best proof in the world but its not a tautology as Rumborak suggested.

To elaborate on my point: In the end, 0.999... = 1 makes a statement about the infinitely small remainder between the two numbers, i.e. that the remainder *is* indeed zero and they are thus the same numbers.
Now, 1/9 = 0.1111.... makes exactly the same point! Just as in 0.9999... the argument goes there is always another "9" that makes it closer to 1, in 0.1111... there is always another 1 that makes it closer to 1/9.

So, my point is that you are starting out with the thing you are actually trying to prove, and then, duh, you magically prove it!

Again, I do not question the overall truth of the statement, I'm just commenting on that a lot of those "easy proofs" for this are inherently flawed.

rumborak

Then what is the "real proof"? I think we proved it last semester in one of my courses called "Proving and reasoning" using Dedekindsneden, whatever that means in English.. Dedekind cuts?

There's many of them, https://en.wikipedia.org/wiki/0.999... has a lot of them, including Dedekind's cut.

rumborak
Title: Re: 48÷2(9+3) =
Post by: 7StringedBeast on April 18, 2011, 08:21:35 AM
Rumby, the thing is, we KNOW that 1/9 = .11111... or .1 with the bar over it. It is a repeating decimal. Then, since we know that, by multiplying .11111....by 9, we can logically conclude that all those 1's turn into 9's, because (1)(9) = 9.

Err, no, James, we DON'T know that 1/9 = .11111..., because it is just a rewording of what you are trying to prove!! (that 0.9999... = 1)
A tautology is not a proof.

rumborak


How about
x = 0.99...
10x = 9.99...
9x = 9.99... - x
9x = 9
x = 1

Correct me if I am wrong but, you can't just subtract x from one side of your equation without also subtracting it from the other.  you should end up with 8x=9  with x being = to 8/9  not 1.
Title: Re: 48÷2(9+3) =
Post by: kári on April 18, 2011, 08:23:02 AM
Well it definitely is the correct notation for that fraction. I think Rumborak's point was that the proof was relying on two unproved axioms, each a different wording of the same problem, whereas in the case of 1/9, the answer of 0.111... can be found using simple mathematics (long division). Admittedly its not the best proof in the world but its not a tautology as Rumborak suggested.

To elaborate on my point: In the end, 0.999... = 1 makes a statement about the infinitely small remainder between the two numbers, i.e. that the remainder *is* indeed zero and they are thus the same numbers.
Now, 1/9 = 0.1111.... makes exactly the same point! Just as in 0.9999... the argument goes there is always another "9" that makes it closer to 1, in 0.1111... there is always another 1 that makes it closer to 1/9.

So, my point is that you are starting out with the thing you are actually trying to prove, and then, duh, you magically prove it!

Again, I do not question the overall truth of the statement, I'm just commenting on that a lot of those "easy proofs" for this are inherently flawed.

rumborak

Then what is the "real proof"? I think we proved it last semester in one of my courses called "Proving and reasoning" using Dedekindsneden, whatever that means in English.. Dedekind cuts?

There's many of them, https://en.wikipedia.org/wiki/0.999... has a lot of them, including Dedekind's cut.

rumborak

I see, thanks. It also includes the proof I gave a few posts above... Does it also make use of the same tautology you are referring to?

Rumby, the thing is, we KNOW that 1/9 = .11111... or .1 with the bar over it. It is a repeating decimal. Then, since we know that, by multiplying .11111....by 9, we can logically conclude that all those 1's turn into 9's, because (1)(9) = 9.

Err, no, James, we DON'T know that 1/9 = .11111..., because it is just a rewording of what you are trying to prove!! (that 0.9999... = 1)
A tautology is not a proof.

rumborak


How about
x = 0.99...
10x = 9.99...
9x = 9.99... - x
9x = 9
x = 1

Correct me if I am wrong but, you can't just subtract x from one side of your equation without also subtracting it from the other.  you should end up with 8x=9  with x being = to 8/9  not 1.
You are correct, but I did subtract x.. 10x - x = 9x.
Title: Re: 48÷2(9+3) =
Post by: 7StringedBeast on April 18, 2011, 08:26:33 AM
Sorry actually we are both wrong.

you subtract x from both sides.

so you have (10x) -x = 9.99 -x

so 10(x) - x = 9

You can't subtract an x out of that multiplication.

EDIT:  Never mind.  I seemed to have forgotten that 10x is actually x + x + x + x etc.  lol I've been out of math classes for far too long.
Title: Re: 48÷2(9+3) =
Post by: kári on April 18, 2011, 09:05:31 AM
:lol What you said above your last line made no sense at all.
Title: Re: 48÷2(9+3) =
Post by: rumborak on April 18, 2011, 09:06:03 AM
I see, thanks. It also includes the proof I gave a few posts above... Does it also make use of the same tautology you are referring to?

It does seem to rely on the same thing, yeah. The key step is when you subtract 0.9999... from 9.99999...., where you are in the same kind of business of working with this incomplete notation. Even though, in that case it's not as blatant as the other one I find.

rumborak
Title: Re: 48÷2(9+3) =
Post by: kári on April 18, 2011, 09:10:48 AM
Really? I would've thought that it's no problem to write 9.99... as 9 + 0.99...
I thought the "key step" was 10*(0.99...) = 9.99...
Title: Re: 48÷2(9+3) =
Post by: tjanuranus on April 18, 2011, 10:18:56 AM
i can't believe the wrong answer is leading this poll WTF.
Title: Re: 48÷2(9+3) =
Post by: mizzl on April 18, 2011, 10:32:41 AM
i can't believe the wrong answer is leading this poll WTF.
This xD
Title: Re: 48÷2(9+3) =
Post by: XJDenton on April 18, 2011, 10:37:12 AM
To be honest I got it wrong before I looked up why. I would never write the equation like that so preference of operations rarely comes up for me.
Title: Re: 48÷2(9+3) =
Post by: rumborak on April 18, 2011, 10:38:11 AM
That's why I voted for "no definite answer". It's still bad syntax.

rumborak
Title: Re: 48÷2(9+3) =
Post by: Scard on April 18, 2011, 12:17:48 PM
In order of operations, division trumps multiplication. Always do the division first.
Title: Re: 48÷2(9+3) =
Post by: rumborak on April 18, 2011, 12:30:45 PM
It's always great to see someone come in and post something that has been shown to be wrong in the previous posts. Multiplication and division are of equal precedence, and you must disambiguate with brackets.

rumborak
Title: Re: 48÷2(9+3) =
Post by: KevShmev on April 18, 2011, 12:33:19 PM
I would think 288 would be correct, but I haven't taken a math class in like eight years, so I am going from memory as far as order of operations.
Title: Re: 48÷2(9+3) =
Post by: lonestar on April 18, 2011, 02:31:44 PM
 :rollin

Still at it, eh guys?
Title: Re: 48÷2(9+3) =
Post by: Dimitrius on April 18, 2011, 02:52:26 PM
:rollin

Still at it, eh guys?
We're a prog forum, aren't we? :lol
Title: Re: 48÷2(9+3) =
Post by: Jamesman42 on April 18, 2011, 04:45:50 PM
Well, rumby, I will just disagree with you on that 0.999 business. But I really don't feel like arguing it

I asked some people today in the math field, they all initially said 2, and when I asked "Are you sure?", most changed their mind and came to 288.
Title: Re: 48÷2(9+3) =
Post by: Orbert on April 18, 2011, 05:18:49 PM
I see the 2(9+3) as one term, so the implied multiplication does in effect take precedence.

For example, if it was 48÷2x, you would definitely simplify 2x first, then divide 48 by the result.  It's not 48÷2*x.  The implied multiplication isn't the same in hierarchy, it comes in with higher priority.  It never even occurred to me to go left to right, because there's no multiplication sign there.  You divide by 2(9+3).

So the answer is 2.  Asking someone "Are you sure?" isn't just asking if they're sure; it makes them think that you know they're wrong and you're giving them another chance to get it right, so they switch to the only other possibility.
Title: Re: 48÷2(9+3) =
Post by: kári on April 18, 2011, 05:23:26 PM
According to the rules of math, the answer is 288. It's as simple as that. Whether it looks messed up or not, it's 288.
Title: Re: 48÷2(9+3) =
Post by: LieLowTheWantedMan on April 18, 2011, 05:24:54 PM
According to the rules of math, the answer is 288. It's as simple as that. Whether it looks messed up or not, it's 288.
Title: Re: 48÷2(9+3) =
Post by: ScioPath on April 18, 2011, 05:35:04 PM
Re: the 0.999... snafu:

Asking whether or not 0.999... = 1 is the same as asking whether 1/∞ = 0.

1 - 0.999... = 0.000...1
=10-∞
=1/10
=1/∞
IFF 1/∞ = 0, then 0.999... = 1.

The problem is, 1/∞ is debatable. One can say the answer is zero, an infinitesimally small number, or undefinable.

My answer is: yes, 0.99... is equal to 1, but only because we need it to be.
Title: Re: 48÷2(9+3) =
Post by: XJDenton on April 18, 2011, 05:53:39 PM
Re: the 0.999... snafu:

Asking whether or not 0.999... = 1 is the same as asking whether 1/∞ = 0.

1 - 0.999... = -0.000...1

Pretty sure you can't do that.
Title: Re: 48÷2(9+3) =
Post by: Jamesman42 on April 18, 2011, 05:58:30 PM
So the answer is 2.  Asking someone "Are you sure?" isn't just asking if they're sure; it makes them think that you know they're wrong and you're giving them another chance to get it right, so they switch to the only other possibility.

But to me, math is concrete. Me asking them makes them look at it deeper and discover that there may be more to it. So I interjected a little doubt to their answer; I didn't say theirs was wrong. They all switched because they reasoned it out like I did in the end.


The problem is, 1/∞ is debatable. One can say the answer is zero, an infinitesimally small number, or undefinable.

My answer is: yes, 0.99... is equal to 1, but only because we need it to be.

In calculus, when we take limits for a variable approaching infinity, we do say that 1/∞ = 0. Just like 0.999 = 1, that is just what it is.

Title: Re: 48÷2(9+3) =
Post by: Octawakeimages on April 18, 2011, 06:16:55 PM
I say 2.  Yay for PEMDAS!  PLEASE excuse my dear aunt sally!!!    :biggrin:

Title: Re: 48÷2(9+3) =
Post by: slycordinator on April 18, 2011, 06:37:04 PM
I see the 2(9+3) as one term, so the implied multiplication does in effect take precedence.
So they'd be separate terms if I made it "2*(9+3)"?

And this seems like a circular argument. Why is it one term? Because implied multiplication takes precedence. Why does implied multiplication take precedence? Because I see it as one term.
Title: Re: 48÷2(9+3) =
Post by: Quadrochosis on April 18, 2011, 06:54:41 PM
48 / 2(9+3)
48 / ((2x9)+(2x3))
48 / (18 + 6)
48 / (24)
2

Where is the difficulty?
Title: Re: 48÷2(9+3) =
Post by: lonestar on April 18, 2011, 07:01:53 PM
I hearby declare both 2 and 288 to be correct, if only to save my sanity.
Title: Re: 48÷2(9+3) =
Post by: bloop on April 18, 2011, 07:03:21 PM
Sorry, that's not how the internet works.
Title: Re: 48÷2(9+3) =
Post by: slycordinator on April 18, 2011, 07:06:07 PM
48 / 2(9+3)
48 / ((2x9)+(2x3))
48 / (18 + 6)
48 / (24)
2

Where is the difficulty?
48 / 2(9+3)
48 / 2 * (9+3)
48 / 2 * (12)
24 * 12
288

Where is the difficulty?
Title: Re: 48÷2(9+3) =
Post by: LieLowTheWantedMan on April 18, 2011, 07:06:19 PM
I say 2.  Yay for PEMDAS!  PLEASE excuse my dear aunt sally!!!    :biggrin:


Why is it no one seems to know how to do PEMDAS/BEDMAS?
Title: Re: 48÷2(9+3) =
Post by: Jamesman42 on April 18, 2011, 07:09:33 PM
Where is the difficulty?

For the nth time, division and multiplication are in the same rank.
Title: Re: 48÷2(9+3) =
Post by: Dimitrius on April 18, 2011, 07:11:38 PM
Where is the difficulty?

For the nth time, division and multiplication are in the same rank.
Yes, but in the syntax that Quad wrote it, it is 2.
Title: Re: 48÷2(9+3) =
Post by: Jamesman42 on April 18, 2011, 07:13:54 PM
Where is the difficulty?

For the nth time, division and multiplication are in the same rank.
Yes, but in the syntax that Quad wrote it, it is 2.

Oh, so he did a separate problem. Gotcha.
Title: Re: 48÷2(9+3) =
Post by: The Degenerate on April 18, 2011, 07:17:54 PM
It's 288. Regardless of PEMDAS/BEDMAS, multiplication and division are equal in the order of operations. So after doing the parenthesis, you go left to right.
Title: Re: 48÷2(9+3) =
Post by: ScioPath on April 18, 2011, 08:13:17 PM
Re: the 0.999... snafu:

Asking whether or not 0.999... = 1 is the same as asking whether 1/∞ = 0.

1 - 0.999... = 0.000...1

Pretty sure you can't do that.

Of course you can, it just reduces the argument. The 0.000...1 either represents an infinitely small number or zero, which is exactly what we are trying to determine. People who argue that 0.999... is not equal to 1 imply that 0.000...1 is in fact a number of measurable substance greater than zero, while those who argue that 0.999... is equal to 1 imply that 0.000...1 is zero. I do not agree that a number approaching a number, even to the point of being infinitely similar, can ever be equal to the number.
Title: Re: 48÷2(9+3) =
Post by: Gorille85 on April 18, 2011, 08:47:25 PM
It's 37.
Title: Re: 48÷2(9+3) =
Post by: Nic35 on April 18, 2011, 08:51:01 PM
It's 37.
:|
Title: Re: 48÷2(9+3) =
Post by: lonestar on April 18, 2011, 08:53:47 PM
It's 37.
:|
Title: Re: 48÷2(9+3) =
Post by: Orbert on April 18, 2011, 10:53:31 PM
I see the 2(9+3) as one term, so the implied multiplication does in effect take precedence.
So they'd be separate terms if I made it "2*(9+3)"?

And this seems like a circular argument. Why is it one term? Because implied multiplication takes precedence. Why does implied multiplication take precedence? Because I see it as one term.

If it said 48÷2x = ?  would you divide by 2, then multiply by x?  No, you would find 2x and divide by the result.  Why is 48÷2(9+3) any different?
Title: Re: 48÷2(9+3) =
Post by: MetalManiac666 on April 18, 2011, 10:55:03 PM
288 all up in this bitch.
Title: Re: 48÷2(9+3) =
Post by: Implode on April 18, 2011, 11:07:57 PM

If it said 48÷2x = ?  would you divide by 2, then multiply by x?  No, you would find 2x and divide by the result.  Why is 48÷2(9+3) any different?

I would ask you to clarify, because the equation is too amboguous.
Title: Re: 48÷2(9+3) =
Post by: rumborak on April 18, 2011, 11:09:29 PM
I see the 2(9+3) as one term, so the implied multiplication does in effect take precedence.
So they'd be separate terms if I made it "2*(9+3)"?

And this seems like a circular argument. Why is it one term? Because implied multiplication takes precedence. Why does implied multiplication take precedence? Because I see it as one term.

If it said 48÷2x = ?  would you divide by 2, then multiply by x?  No, you would find 2x and divide by the result.  Why is 48÷2(9+3) any different?

What about 48÷2(9+3)(5+3)(2+8) ?
Do you just assume everything after the ÷ sign to be in the denominator? That's a pretty arbitrary decision given multiplication and division are of equal rank.

rumborak
Title: Re: 48÷2(9+3) =
Post by: Orbert on April 18, 2011, 11:12:35 PM
Yeah, I would.  I always presumed that implied multiplication gave it some kind of precedence.  But it's not in the formal rules, so I guess not.

But I noticed that no one has answered my 48÷2x question, and it's techinically the same thing.  All you're saying is 48÷2abc.  I would still find 2abc and divide it into 48.  I guess it's because there's no actual multiplication sign in the expression.  It seems like you'd simplify it first.
Title: Re: 48÷2(9+3) =
Post by: Xanthul on April 18, 2011, 11:15:24 PM
Re: the 0.999... snafu:

Asking whether or not 0.999... = 1 is the same as asking whether 1/∞ = 0.

1 - 0.999... = 0.000...1

Pretty sure you can't do that.

Of course you can, it just reduces the argument. The 0.000...1 either represents an infinitely small number or zero, which is exactly what we are trying to determine. People who argue that 0.999... is not equal to 1 imply that 0.000...1 is in fact a number of measurable substance greater than zero, while those who argue that 0.999... is equal to 1 imply that 0.000...1 is zero. I do not agree that a number approaching a number, even to the point of being infinitely similar, can ever be equal to the number.

0.000...1 is not a correct number in any notation. You can't put a "1" at the end of an INFINITE string of zeros. That's the whole reason 0.999... is equal to 1, because there is no number inbetween and no difference between them.

1 - 0.999... = 0.000... = 0
Title: Re: 48÷2(9+3) =
Post by: Ħ on April 18, 2011, 11:46:13 PM
Why is the wrong answer winning.
Title: Re: 48÷2(9+3) =
Post by: rumborak on April 18, 2011, 11:51:06 PM
Why do people still say there is a right answer?

rumborak
Title: Re: 48÷2(9+3) =
Post by: Ħ on April 18, 2011, 11:55:42 PM
multiplication and division are of equal rank
/thread
Title: Re: 48÷2(9+3) =
Post by: 73109 on April 19, 2011, 12:16:27 AM
P
E
M
D
A
S

2 is the right answer.
Title: Re: 48÷2(9+3) =
Post by: Ħ on April 19, 2011, 12:18:36 AM
P
E
M
D
A
S

2 is the right answer.
You are misinterpreting PEMDAS.

P
E
MD (in order of left to right)
AS (in order of left to right)
Title: Re: 48÷2(9+3) =
Post by: MetalManiac666 on April 19, 2011, 12:20:03 AM
I love how people are choosing 2, and then giving PEMDAS as the reason.  It's PEMDAS that shows that 288 is the correct answer.  It's only when you take into account implied multiplication that the answer comes out as 2.
Title: Re: 48÷2(9+3) =
Post by: Implode on April 19, 2011, 12:22:38 AM
I love how people are choosing 2, and then giving PEMDAS as the reason.  It's PEMDAS that shows that 288 is the correct answer.  It's only when you take into account implied multiplication that the answer comes out as 2.
Title: Re: 48÷2(9+3) =
Post by: The Degenerate on April 19, 2011, 01:08:22 AM
P
E
M
D
A
S

2 is the right answer.
You are misinterpreting PEMDAS.

P
E
MD (in order of left to right)
AS (in order of left to right)

THIS. The only people saying 2 are those that mistakingly believe that multiplication takes priority over division in all equations, which is incorrect.
Title: Re: 48÷2(9+3) =
Post by: kári on April 19, 2011, 01:22:10 AM
Re: the 0.999... snafu:

Asking whether or not 0.999... = 1 is the same as asking whether 1/∞ = 0.

1 - 0.999... = 0.000...1
No. Please go learn some more math so you know what you are talking about.
Also, 1/∞ = 0. Not infinitesimal, not undefined, it's 0.

Why do people still say there is a right answer?

rumborak
Because there is... Regardless of whether you think it is bad syntax or not, the right answer is 288.
Title: Re: 48÷2(9+3) =
Post by: jsem on April 19, 2011, 02:34:17 AM
How long does this have to go on? Lol.

Move to P/R? :neverusethis:
Title: Re: 48÷2(9+3) =
Post by: ScioPath on April 19, 2011, 04:51:25 AM
Re: the 0.999... snafu:

Asking whether or not 0.999... = 1 is the same as asking whether 1/∞ = 0.

1 - 0.999... = 0.000...1
No. Please go learn some more math so you know what you are talking about.
Also, 1/∞ = 0. Not infinitesimal, not undefined, it's 0

"Learn math or gtfo." :clap: brilliant constructive answer. Kari, I think you misunderstand that what I posted is entirely hypothetical. What people are implying when they say 1 is not equal to 0.999... is that there is an infinitessimal difference between it and 1. I just took the problem and rephrased it to better state the problem at hand. Why am I getting so much "derp, you can't do that," crap?

Also, I am a bit confused, are you saying the infinitesimal doesn't exist? If an infinitely large number exists, why doesn't an infinitely small number exist, and why can't they be multiplied to be 1?
Title: Re: 48÷2(9+3) =
Post by: XJDenton on April 19, 2011, 05:33:18 AM
The notation you used implied an infinite number of zeros with a one on the end, which is nonsensical, as an infinite sequence can not have an end number.
Title: Re: 48÷2(9+3) =
Post by: kári on April 19, 2011, 06:36:56 AM
Re: the 0.999... snafu:

Asking whether or not 0.999... = 1 is the same as asking whether 1/∞ = 0.

1 - 0.999... = 0.000...1
No. Please go learn some more math so you know what you are talking about.
Also, 1/∞ = 0. Not infinitesimal, not undefined, it's 0

"Learn math or gtfo." :clap: brilliant constructive answer. Kari, I think you misunderstand that what I posted is entirely hypothetical. What people are implying when they say 1 is not equal to 0.999... is that there is an infinitessimal difference between it and 1. I just took the problem and rephrased it to better state the problem at hand. Why am I getting so much "derp, you can't do that," crap?

Also, I am a bit confused, are you saying the infinitesimal doesn't exist? If an infinitely large number exists, why doesn't an infinitely small number exist, and why can't they be multiplied to be 1?
That wasn't what I was saying, I was trying to say that what you wrote is entirely incorrect. You're confusing the set of real numbers with some extension of it that includes infinitesimal numbers. But that was never the point. The point is that in the set of real numbers, 0.99.. = 1.

Also in the set of real numbers, 1/∞ = 0. Infinitesimal numbers "exist", but not in the set real numbers. ∞ is also not an element of the set of real numbers, that's why, when working with it, certain operations have been exactly defined. 1/∞ = 0 because lim(x->∞)(1/x) = 0.

Also, writing the infinitesimal difference between 1/∞ and 0 in that extended set of the reals as 0.00...1 makes no sense as well. There is no 1 at the end, since there is no end. There's no way to write an infinitesimal number like you would write another real number. Just like you can't write infinity, which is why the symbol ∞ was introduced. What you mean when you say that 1/∞ is infinitesimal is that 1/∞ is as close as you can get to 0, without writing 0. It has no meaning however.
Title: Re: 48÷2(9+3) =
Post by: robwebster on April 19, 2011, 06:38:16 AM
The notation you used implied an infinite number of zeros with a one on the end, which is nonsensical, as an infinite sequence can not have an end number.
That's the exact point of the debate, though, surely? Whether there's a difference between something that's infinitesimally small, and something that doesn't exist.

I'd say the difference between the two is nil, and I'd therefore say that nought point nine-with-a-dot-above-it is equal to 1. I agree with you.  But it's not a foregone conclusion, I don't think. I wouldn't tell anyone who disagrees with me to learn more maths. Poss to reconsider, but I get where ScioPath was coming from. The point of the ... isn't saying that the series ends, rather to demonstrate that the eventual "...01" is always, always a step further away.
Title: Re: 48÷2(9+3) =
Post by: Orbert on April 19, 2011, 07:36:18 AM
The only people saying 2 are those that mistakingly believe that multiplication takes priority over division in all equations, which is incorrect.

No.  There are some who were taught that implied multiplication pre-empts normal left-to-right precedence and have yet to hear any counterargument.  It is not the same issue as order of operations.
Title: Re: 48÷2(9+3) =
Post by: kári on April 19, 2011, 07:38:22 AM
The notation you used implied an infinite number of zeros with a one on the end, which is nonsensical, as an infinite sequence can not have an end number.
That's the exact point of the debate, though, surely? Whether there's a difference between something that's infinitesimally small, and something that doesn't exist.

I'd say the difference between the two is nil, and I'd therefore say that nought point nine-with-a-dot-above-it is equal to 1. I agree with you.  But it's not a foregone conclusion, I don't think. I wouldn't tell anyone who disagrees with me to learn more maths. Poss to reconsider, but I get where ScioPath was coming from. The point of the ... isn't saying that the series ends, rather to demonstrate that the eventual "...01" is always, always a step further away.
I wasn't trying to be rude or so, I was just trying to say that in order to really know what you are talking about you have to know some stuff about certain branches of math. If you don't, you don't really have much ground to stand on. This is not like the original topic of this thread where anyone can participate as the problem is mundane.

It would be like you or me joining some quantum physicists in a discussion about quantum entanglement and go "Hey, why are you debating this? It can't be possible for something to have an effect on something else without anything in between knowing about it". Or astrophysicists discussion black hole radiation and saying "Black holes can't emit stuff, nothing can escape them, not even light."
Title: Re: 48÷2(9+3) =
Post by: Jamesman42 on April 19, 2011, 08:27:22 AM
No.  There are some who were taught that implied multiplication pre-empts normal left-to-right precedence and have yet to hear any counterargument.  It is not the same issue as order of operations.

^Turns out this may be true: https://mathforum.org/library/drmath/view/54341.html

Although the most common way is to do it the way to get 288, not 2. I still stick with 288, but I am more with rumby that it's just bad syntax, and I agree with the above article that parentheses are definitely a plus to make sure there is no ambiguity. Parentheses are an expression's best friend, and they can be yours, too.

I was taught left to right and not the implied multiplication (never heard of IM until this thread).
Title: Re: 48÷2(9+3) =
Post by: kári on April 19, 2011, 08:37:04 AM
Same here James. Either way, it's not a standard form, nor should it be. Parentheses should be used.
Title: Re: 48÷2(9+3) =
Post by: Orbert on April 19, 2011, 10:56:38 AM
(never heard of IM until this thread)

Me neither, at least not as a formalized term.
Title: Re: 48÷2(9+3) =
Post by: lonestar on April 19, 2011, 01:01:59 PM
Day 3, and the battle still rages. :rollin
Title: Re: 48÷2(9+3) =
Post by: Orbert on April 19, 2011, 01:16:14 PM
Awesome, isn't it?
Title: Re: 48÷2(9+3) =
Post by: lonestar on April 19, 2011, 01:23:13 PM
It's especially entertaining to me because I am worthless at math.  It would be like you guys watching me and ten other chefs arguing whether sous vide or slow roasting garners a better duck breast.
Title: Re: 48÷2(9+3) =
Post by: jsem on April 19, 2011, 02:03:30 PM
It's especially entertaining to me because I am worthless at math.  It would be like you guys watching me and ten other chefs arguing whether sous vide or slow roasting garners a better duck breast.
I don't even know what you're talking about...
Title: Re: 48÷2(9+3) =
Post by: lonestar on April 19, 2011, 02:34:00 PM
It's especially entertaining to me because I am worthless at math.  It would be like you guys watching me and ten other chefs arguing whether sous vide or slow roasting garners a better duck breast.
I don't even know what you're talking about...
My point exactly.
Title: Re: 48÷2(9+3) =
Post by: Orbert on April 19, 2011, 03:27:13 PM
Did someone say "breast"?
Title: Re: 48÷2(9+3) =
Post by: Jamesman42 on April 19, 2011, 03:54:39 PM
Saying "breast" in a thread about math is like giving a 12-year-old boy a copy of Maxim after a dog has torn it up. Such a tease and that's all we are getting.
Title: Re: 48÷2(9+3) =
Post by: XJDenton on April 19, 2011, 04:16:05 PM
The notation you used implied an infinite number of zeros with a one on the end, which is nonsensical, as an infinite sequence can not have an end number.
That's the exact point of the debate, though, surely? Whether there's a difference between something that's infinitesimally small, and something that doesn't exist.

I'd say the difference between the two is nil, and I'd therefore say that nought point nine-with-a-dot-above-it is equal to 1. I agree with you.  But it's not a foregone conclusion, I don't think. I wouldn't tell anyone who disagrees with me to learn more maths. Poss to reconsider, but I get where ScioPath was coming from. The point of the ... isn't saying that the series ends, rather to demonstrate that the eventual "...01" is always, always a step further away.

As a conceptual argument that makes sense but he used the notion in a mathematical proof, which simply doesn't work.
Title: Re: 48÷2(9+3) =
Post by: Ħ on April 19, 2011, 05:19:09 PM
Implied multiplication is a lie.

Honestly, division signs and /'s are just amateur.  Real mathematicians use horizontal lines to indicate division.
Title: Re: 48÷2(9+3) =
Post by: XJDenton on April 19, 2011, 05:24:06 PM
Actually I think real mathematicians use whatever symbol their computer code uses, therefore allowing them to bugger off whilst it runs to drink some more coffee.
Title: Re: 48÷2(9+3) =
Post by: Jamesman42 on April 19, 2011, 05:28:29 PM
Honestly, division signs and /'s are just amateur.  Real mathematicians use horizontal lines to indicate division.

Lol?
Title: Re: 48÷2(9+3) =
Post by: rumborak on April 19, 2011, 05:30:59 PM
I think given what real mathematicians works on, 48÷2(9+3) comes out to be 3+0.8i, or something.

rumborak
Title: Re: 48÷2(9+3) =
Post by: Jamesman42 on April 19, 2011, 05:32:59 PM
Or 2222222
Title: Re: 48÷2(9+3) =
Post by: XJDenton on April 19, 2011, 05:37:23 PM
https://en.wikipedia.org/wiki/Graham%27s_number
Title: Re: 48÷2(9+3) =
Post by: Orbert on April 19, 2011, 07:18:44 PM
As many have pointed out, it's really completely pointless.  The rules for simplifying or evaluating an expression are only so that students learning algebra will know how to read and write the shorthand.  In an actual application, it would be pretty obvious whether you're supposed to divide or multiply first to get the correct answer.
Title: Re: 48÷2(9+3) =
Post by: Tomislav95 on April 20, 2011, 10:50:30 AM
I think it's 288 because we have to go one after the other :-\
Title: Re: 48÷2(9+3) =
Post by: slycordinator on April 20, 2011, 08:37:10 PM
And since I was asked, I would in fact evaluate 48÷2x as 24x. I err on the side of assuming that implied multiplication is of equal rank to normal multiplication since I've seen less mathematicians claiming otherwise.
Title: Re: 48÷2(9+3) =
Post by: Jamesman42 on April 20, 2011, 09:05:09 PM
sly, maybe you can help me with this

I was tutoring and a kid asked me this problem:

"Suppose a cylinder full of sugar has a mass of 5.81. When it is 3/8 full, its mass is 3.8. What is the mass of the cylinder when it is empty?"

The thing that is confusing me is that it is mass. I know volume and mass aren't the same thing, so I am unsure of how to approach this problem. Supposed to be an Algebra 1 problem and I am drawing a blank.
Title: Re: 48÷2(9+3) =
Post by: Jamesman42 on April 20, 2011, 09:10:34 PM
No radius or height was given, by the way, which further perplexed me. I hope I'm not missing something obvious.
Title: Re: 48÷2(9+3) =
Post by: Orbert on April 20, 2011, 09:27:12 PM
That is why application problems should always include units.  When units are provided, as they should be, there's no ambiguity.
Title: Re: 48÷2(9+3) =
Post by: Jamesman42 on April 20, 2011, 09:34:36 PM
Let's say the units are in kilograms
Title: Re: 48÷2(9+3) =
Post by: rumborak on April 20, 2011, 10:03:02 PM
Volume and mass are proportional. Whatever unit system you would use doesn't matter, it would cancel out anyway.

rumborak
Title: Re: 48÷2(9+3) =
Post by: Jamesman42 on April 20, 2011, 10:06:23 PM
That is what I figured, but that isn't my issue...how do I solve this? Argh, I feel dumb for not knowing this
Title: Re: 48÷2(9+3) =
Post by: Fiery Winds on April 20, 2011, 10:11:23 PM
That is what I figured, but that isn't my issue...how do I solve this? Argh, I feel dumb for not knowing this

Mass of Sugar = S
Mass of Cylinder = C

S + C = 5.81
(3/8)S + C = 3.8

System of two equations.   :tup  Took me a second too.
Title: Re: 48÷2(9+3) =
Post by: Jamesman42 on April 20, 2011, 10:14:21 PM
*bangs head on keyboard*

I don't understand why I can do all sorts of calculus problems and these system of 2 equations algebra 1 problems always get me. There's like a gap in my knowledge there. I sorta had an idea it was gonna be a system of equations, but it eluded me.

Thank you FW, you rock
Title: Re: 48÷2(9+3) =
Post by: rumborak on April 20, 2011, 10:16:32 PM
"Suppose a cylinder full of sugar has a mass of 5.81.

5.81 = 1.0*C+M0

1.0*C : "1.0 of the volume, times whatever conversion factor into mass"
M0 : Mass of the cylinder

Quote
When it is 3/8 full, its mass is 3.8.

3.8 = (3/8)*C + M0

From equation 1:

C = 5.81 - M0

into equation 2:

3.8 = (2/8)*(5.81-M0) + M0

<=> 3.8-(2/8)*5.81 = (6/8)*M0

<=> M0 = 3.13

Gnaah, ninja edit!!

rumborak
Title: Re: 48÷2(9+3) =
Post by: Implode on April 21, 2011, 01:00:08 AM
Hey guys. 288 is winning. Let's celebrate by me asking another argument inducing question.

So there's a plane on a conveyor belt... :neverusethis:
Title: Re: 48÷2(9+3) =
Post by: kári on April 21, 2011, 02:17:24 AM
It would take off.
Title: Re: 48÷2(9+3) =
Post by: Jamesman42 on April 21, 2011, 08:14:37 AM
Massive text haha get it MASSive haha oh man haha...ha


Man, I was tired last night, looking back I really should have known all that, it being mass doesn't have that much to do with setting up the system of equations...gah!
Title: Re: 48÷2(9+3) =
Post by: adameastment on April 21, 2011, 08:21:11 AM
2? What? The order of operations (https://en.wikipedia.org/wiki/Order_of_operations) says that in this case you do parentheses first and then divide/multiply from left to right.

48÷2(9+3) = ?
48÷2x 12 =?
24x12=?
288
In the UK it's BIDMAS

Brackets first.
Indicies next
Division
Multiplication
Addition
Subtraction...
Title: Re: 48÷2(9+3) =
Post by: 7StringedBeast on April 21, 2011, 08:37:44 AM
But wouldn't that expression be the exact same thing as:

48
___

2(9+3)


So

48
_____    =  2
24


Title: Re: 48÷2(9+3) =
Post by: jsem on April 21, 2011, 08:48:27 AM
But wouldn't that expression be the exact same thing as:

48
___

2(9+3)


So

48
_____    =  2
24



No, it's not the same thing. Check my post a few pages back.
Title: Re: 48÷2(9+3) =
Post by: adameastment on April 21, 2011, 08:53:39 AM
But wouldn't that expression be the exact same thing as:

48
___

2(9+3)


So

48
_____    =  2
24




Yeah, I forgot to add that bit :P

I was gonna add "So it's definitely 2" :P
Title: Re: 48÷2(9+3) =
Post by: 7StringedBeast on April 21, 2011, 08:54:57 AM
To get 288 shouldnt it be (48/2)*(9+3)  Other wise, you write it the way I wrote it above ^^. 
Title: Re: 48÷2(9+3) =
Post by: jsem on April 21, 2011, 08:56:21 AM
I get it nao lol.

I automatically jumped to this conclusion:

    48
______
2(9+3)

In which case it's 2. This is the same as 48 ÷ (2(9+3)). But that was not the given though.


Title: Re: 48÷2(9+3) =
Post by: adameastment on April 21, 2011, 08:58:25 AM
To get 288 shouldnt it be (48/2)*(9+3)  Other wise, you write it the way I wrote it above ^^. 
Yeah, because with the order of operations it goes something like this:

48/2(9+3) = 48/24 = 2
Title: Re: 48÷2(9+3) =
Post by: slycordinator on April 21, 2011, 09:08:57 AM
To get 288 shouldnt it be (48/2)*(9+3)  Other wise, you write it the way I wrote it above ^^.  
No.
1) The equation given is 48/2(9+3). You rewrote this as 48/(2(9+3)) . You can't both say "You can only get 288 if you add some parentheses" while yourself saying that the answer is 2 while implicitly adding them in there yourself.
2) And no. You don't have to put parentheses around the 48/2 term... because multiplication and division are of equal rank and when they both occur together in an equation you do the first one that appears in it while going from left-to-right.
Title: Re: 48÷2(9+3) =
Post by: adameastment on April 21, 2011, 09:11:42 AM
But in school you are told to ALWAYS expand the brackets first?
Title: Re: 48÷2(9+3) =
Post by: slycordinator on April 21, 2011, 09:24:34 AM
But in school you are told to ALWAYS expand the brackets first?
And I never said otherwise. A 2 that isn't in a bracket isn't part of the bracket.

1) 48/2(9+3)
2) 48/2*(9+3)
3) 48/2*(12)
*and here, since multiplication and division are equal rank, I do the first one that appears in left-to-right, which is the division*
4) 24*(12)
5) 288
Title: Re: 48÷2(9+3) =
Post by: Orbert on April 21, 2011, 11:52:12 AM
That's the dichotomy.  I've always thought the the 2 being adjacent to the (9+3) gave it some kind of precedence, but apparently there is no such rule.  So a strict application of the rules yields the above result.